SlideShare una empresa de Scribd logo
1 de 109
contents   next
VISION
   A   premier   university   in   CALABARZON,
offering academic programs and related services
designed to respond to the requirements of the
Philippines and the global economy, particularly
in Asian Countries.

                               contents   back   next
MISSION
      The University shall primarily provide advanced
education, professional, technological and vocational
instruction                                                       in
agriculture, fisheries, forestry, science, engineering, in
dustrial             technologies,                       teacher
education,       medicine,       law,           arts        and
sciences, information technologies and other related
fields. It shall also undertake research and extension
services and provide progressive leadership in its
areas of specialization.             contents     back     next
GOALS
      In pursuit of the college vision/mission the
College of Education is committed to develop the
full potentials of the individuals and equip them
with knowledge, skills and attitudes in Teacher
Education allied fields to effectively respond to the
increasing demands, challenges and opportunities
of changing time for global competitiveness.


                                  contents   back   next
OBJECTIVES OF BSED
             Produce graduate who can demonstrate and practice the
professional and ethical requirement for the Bachelor of Secondary
Education such as:

1.To serve as positive and powerful role models in the pursuit of the learning thereby
maintaining high regards to professional growth.
2. Focus on the significance of providing wholesome and desirable learning environment.
3. Facilitate learning process in diverse type of learners.
4. Used varied learning approaches and activities, instructional materials and learning
resources.
5. Used assessment data, plan and revise teaching – learning plans.
6. Direct and strengthen the links between school and community activities.
7. Conduct research and development in Teacher Education and other related activities.


                                                              contents   back   next
This Teacher’s “MODULE IN SOLVING QUADRATIC EQUATION” is part of the requirements in
Educational Technology 2 under the revised curriculum based on CHED Memorandum Order (CMO)-30, Series
of 2004. Educational Technology 2 is a three (3)-unit course designed to introduce both traditional and innovative
technologies to facilitate and foster meaningful and effective learning where students are expected to
demonstrate a sound understanding of the nature, application and production of the various types of educational
technologies.

        Students are provided with guidance and assistance of selected faculty The members of the College on
the selection, production and utilization of appropriate technology tools in developing technology-based teacher
support materials. Through the role and functions of computers especially the Internet, the student researchers
and the advisers are able to design and develop various types of alternative delivery systems. These kinds of
activities offer a remarkable learning experience for the education students as future mentors especially in the
preparation and utilization of instructional materials.

       The output of the group’s effort may serve as a contribution to the existing body instructional materials
that the institution may utilize in order to provide effective and quality education. The lessons and evaluations
presented in this module may also function as a supplementary reference for secondary teachers and students.
reference for secondary teachers and students.


                                                                                  Aleli M. Ariola
                                                                                  Module Developer


                                                                                  Shane Maureen D. Atendido
                                                                                  Module Developer


                                                                          contents           back         next
This Teacher’s “MODULE IN SOLVING QUADRATIC EQUATION” is part of the requirements in Educational
Technology 2 under the revised curriculum based on CHED Memorandum Order (CMO)-30, Series of 2004.
Educational Technology 2 is a three (3)-unit course designed to introduce both traditional and innovative technologies
to facilitate and foster meaningful and effective learning where students are expected to demonstrate a sound
understanding of the nature, application and production of the various types of educational technologies.

         Students are provided with guidance and assistance of selected faculty The members of the College through
the selection, production and utilization of appropriate technology tools in developing technology-based teacher
support materials. Through the role and functions of computers especially the Internet, the student researchers and
the advisers are able to design and develop various types of alternative delivery systems. These kinds of activities
offer a remarkable learning experience for the education students as future mentors especially in the preparation and
utilization of instructional materials.

         The output of the group’s effort may serve as a contribution to the existing body instructional materials that the
institution may utilize in order to provide effective and quality education. The lessons and evaluations presented in this
module may also function as a supplementary reference for secondary teachers and students. reference for secondary
teachers and students.
                                                                                     FOR-IAN V. SANDOVAL
                                                                                     Computer Instructor / Adviser
                                                                                     Educational Technology 2

                                                                                    DELIA F. MERCADO
                                                                                    Module Consultant / Instructor 3
                                                                                    Principal of Laboratory High School

                                                                                    LYDIA R. CHAVEZ
                                                                                    Dean College of Education

                                                                               contents              back         next
The authors would like to acknowledge with deep appreciation and gratitude the invaluable help of the following
persons:

      Mr. For-Ian V. Sandoval our module adviser, Computer Instructor / Adviser Educational Technology 2 for giving us
opportunity to participate on this project, and for guiding us and pursue us to finish this module.

      Mrs. Delia F. Mercado, Instructor III and Director of Laboratory High School, for being our Teacher Consultant for
the completion of this modular workbook.

           Mrs. Corazon San Agustin, our instructor in Educational Technology I, for giving us guidance and
encouragement us in completing the requirement.

      Mrs. Lydia Chavez, Dean of Education for the support and guidance.

      We also wish to thank our family and friends as an inspiration and understand us they were robbed of many
precious moments as we looked ourselves in our rooms when our minds went prolific and our hands itched to write.

       And finally, we thank Almighty God, the source of all knowledge, understanding and wisdom. From him we owe all
that we have and all that we are!

      Once again, we thank all those who have encourage and helped us in preparing this module for publication and
who have extended us much understanding, patience, and support.

                                                                                                 THE AUTHORS


                                                                             contents            back        next
A quadratic equation is a second-order polynomial equation in a single variable x.

      The general form is                             where x represents a variable, and a, b, and c, represent coefficients
and constants, with a ≠ 0. (If a = 0, the equation becomes a linear equation.)

      Among his many other talents, Major General Stanley in Gilbert and Sullivan's operetta the Pirates of Penzance
impresses the pirates with his knowledge of quadratic equations in "The Major General's Song" as follows: "I am the very
model of a modern Major-General, I've information vegetable, animal, and mineral, I know the kings of England, and I
quote the fights historical, From Marathon to Waterloo, in order categorical; I'm very well acquainted too with matters
mathematical, I understand equations, both the simple and quadratic, About binomial theorem I'm teeming with a lot o'
news-- With many cheerful facts about the square of the hypotenuse."

       The constants a, b, and c, are called respectively, the quadratic coefficient, the linear coefficient and the constant
term or free term. Quadratic comes from quadratus, which is the Latin word for "square." Quadratic equations can be
solved by factoring, completing the square, graphing, Newton's method, and using the quadratic formula. One common
use of quadratic equations is computing trajectories in projectile motion.

              This module centers on the different ways of solving the quadratic equation by factoring, by finding square
roots, by completing the square, and by using the quadratic formula. Students are given guides to determine the most
appropriate method to use. Identifying the disciminant of the quadratic equation and finding the relationship between the
coefficient and the root of the quadratic equation are also discussed.



                                                                               contents           back        next
At the end of this module, students are expected to:

      •distinguish what is quadratic equation and complex numbers;

      •recognize property of the number I and solutions in solving the four arithmetic operations;

      •solve quadratic equation by factoring, completing the square, quadratic formula and solving by graphing;

      •learn the technique on how to use any method for solving quadratic equation, taking the square root and
      transforming quadratic equations by appropriate substitution;

      •determine discriminant, and relations between roots and coefficient; and

      •use calculator in solving quadratic equation.




                                                                            contents            back     next
VMGO’s of BSEd
Foreword
Acknowledgement
Introduction
General Objective’s
Table of Contents

Chapter I. Identify the Quadratic Equation
    Lesson 1. Quadratic Equation
Chapter II. Complex Number
    Lesson 2. Defining Complex Number
    Lesson 3. Number i
    Lesson 4. Complex Plane
    Lesson 5. Complex Arithmetic
Chapter III. Solving Quadratic Equation
    Lesson 6. Factoring
    Lesson 7. Completing the Square
    Lesson 8. Quadratic Formula
    Lesson 9. Solving by Graphing




                                             contents   back   next
Chapter IV. Solving Equation on Quadratic
    Lesson 10. Equation in Quadratic Form
    Lesson 11. Equation Containing Radicals
    Lesson 12. Equation Reducible to Quadratic Equation
Chapter V. The Discriminant, Roots and Coefficient
    Lesson 13. Discriminant and the Roots of a Quadratic Equation
    Lesson 14. Relation between Roots and Coefficient
Chapter VI. Solving Quadratic Equation on a Calculator
    Lesson 15. Equation on a Calculator

References

Demo
Slide share




                                                         contents   back   next
Chapter I




        This chapter deals with equations which are classified according to the highest power of its variable. An
 equation in the variable x whose highest power is 2 is called a quadratic equation. It will be observed here that
 variable a, b and c are real numbers and a cannot be 0.

   TARGET SKILLS:
                At the end of this chapter, students are expected to:
                       • identify quadratic equation;
                       • discuss real numbers and standard form of the quadratic equation;
                       • express quadratic equation in standard form; and
                       • apply distributive property in solving quadratic equation.


                                                                          contents           back   next
Lesson 1
                              Identifying the quadratic equation
  OBJECTIVES:
               At the end of this lesson, students are expected to:
                      •define the quadratic equation;
                      •discuss real numbers in quadratic equation; and
                      •improve writing the standard form of the quadratic equation.


        Polynomials are classified according to the highest power of its variable. A first degree polynomial, like 2x + 5
is linear; a second degree polynomial, like x2 + 2 – 3 is quadratic; a third degree polynomial, like x3 + 4x2 – 3x + 12 is
cubic.
        Similarly, equation and inequalities are classified according to the highest power of its variable. An equation in
the variable x whose highest power is two is called a quadratic equation. Some examples are x2 – 64, 4n2 = 25, 3x2
– 4x + 1 = 0.
              An equation of the form ax2 + bx + c = 0, where a, b and c are constant and a not equal to 0, a id a
 quadratic equation. to 0, a id a quadratic equation.

              Any quadratic equation can be written in the form ax2 + bx + c = 0. This is also called the standard
form of the quadratic equation. Here, a, b and c are real numbers and a cannot be 0.

Example A. Express x2 = 8x in standard form
           x2 = 8x can be written as x2 - 8x = 0
           where a=1, b= ˉ8, and c=0.

Example B. Express x2 = 64 in standard form



                                                                            contents            back        next
x2 = 64 can be written as x2 – 64 = 0
                where a=1. B=0, and c=ˉ64.

Example C. Express the fractional equation x = 1/x-3 as a quadratic equation.
           x = 1/x-3
           x (x-3) = 1                     multiply both sides by x-3
           x² - 3x = 1                      using the distributive property
           x² - 3x - 1 = 0                  a=1, b=ˉ3, c=ˉ1

Exercises:
Which of the following equations are quadratic?

1.3x = x² - 5
2. 2x =1
3. x² = 25
4. 2x - 3 = x + 5
5. 5x – 2y = 0




                                                                        contents   back   next
Name: ___________________                                             Section: _______
  Instructor: ________________                                          Date: _______                Rating: _______




Instruction: Write the following equations in the form ax2 + bx + c = 0, and give the value of a, b, and c.

1. x2 = 6x
              _____________________________________________

2. 2x2 = 32
              _____________________________________________

3. 3x2 = 5x – 1
              _____________________________________________

4. 10 = 3x – x2
              _____________________________________________

5. (x + 2)2 = 9
               _____________________________________________

6. 4x2 = 64
              _____________________________________________




                                                                            contents            back          next
7.
                  _____________________________________________

8.
                  _____________________________________________

9.    8x = x2
                  _____________________________________________
         2
10.          =6
                  _____________________________________________

11.
                  _____________________________________________
12. x2 =
                  _____________________________________________

13.
                  _____________________________________________

14. x2 +
                  _____________________________________________

15. (x + 1)(x-3) = 6
                  _____________________________________________




                                                           contents   back   next
A. Define each of the following terms.
       1. Quadratic equation
       2. Standard form of a quadratic equation
       3. Real numbers
B. Which of the following equations are quadratic?
       1. 4x = 2x2 – 6
       2. 3x = 1
       3. 5x2 = 30
       4. 3x – 2 = 2x + 6
       5. 2x – 5y = 0
       6. 4x + 2x2 – 3x3 = 0
       7. 12x2 – x = 11
C. Write the following equations in the form ax2 + bx + c = 0, and give the values of a, b and c.
       1. 3x2 = 6x
       2. 3x2 = 32
       3. 2x2 = 5x – 1
       4. 12 = 4x – x2
       5. (x + 3)2 = 8
       6. 4x2 = 56
       7. 1/x + x = 6
       8. x(x – 4) – 1 = 0
       9. 9x = x2
       10. (1/x)2 = 10


                                                                                contents            back   next
Chapter II




        This chapter centers on the complex numbers which is a number comprising a real number and an imaginary
 number. Under this, we have the number i, the complex plane where the points are plotted and the 4 arithmetic
 operations such as addition and subtraction, multiplication and division of complex numbers. To round up the
 chapter, simple equation involving complex numbers will be studied and solved.


     TARGET SKILLS:
           At the end of this chapter, students are expected to:

                 • identify complex numbers;
                 • differentiate the real pat and imaginary part of complex numbers; and
                 • explore solving of the 4 arithmetic operations on the complex numbers.




                                                                         contents           back   next
Lesson 2
                                      Defining Complex Numbers
    OBJECTIVES:

    At the end of this lesson, students are expected to:

                  •identify complex numbers;
                  •differentiate the real number and standard imaginary unit; and
                  •extend the ordinary real number.


       A complex number, in mathematics, is a number comprising a real number and an imaginary number; it can
be written in the form a + bi, where a and b are real numbers, and i is the standard imaginary unit, having the
property that i2 = −1. The complex numbers contain the ordinary real numbers, but extend them by adding in extra
numbers and correspondingly expanding the understanding of addition and multiplication.
       Equation 1: x2 - 1 = 0.

         Equation 1 has two solutions, x = -1 and x = 1. We know that solving an equation in x is equivalent to finding
 the x-intercepts of a graph; and, the graph of y = x2 - 1 crosses the x-axis at (-1,0) and (1,0).




                                                                            contents            back       next
Equation 2: x2 + 1 = 0

             Equation 2 has no solutions, and we can see this by looking at the graph of y = x2 +
1.




                 Since the graph has no x-intercepts, the equation has no solutions. When we define complex
     numbers, equation 2 will have two solutions.




                                                                             contents           back     next
Name: ___________________                        Section: _______
Instructor: ________________                     Date: ________                Rating: _______




 Solve each equation and graph.


 1. x² + 4 = 0
                 _____________________________________________

 2. 2x² + 18 = 0
              _____________________________________________

 3. 2x² + 14 = 0
              _____________________________________________

 4. 3x² + 27 = 0
              _____________________________________________

 5. x² - 3 = 0
                 _____________________________________________

 6. x² + 21 = 0
                 _____________________________________________




                                                             contents   back     next
7. 3x² - 5 = 0
              _____________________________________________

8. 5x² + 30 = 0
             _____________________________________________

9. 2x² + 3 = 0
              _____________________________________________

10. x² + 50 = 0
             _____________________________________________

11. x² - 2 = 0
                 _____________________________________________

12. 3x² - 50 = 0
             _____________________________________________

13. x² - 3 = 0
              _____________________________________________

14. x² + 4 = 0
              _____________________________________________

15. 2x² + 14 = 0
             _____________________________________________




                                                            contents   back   next
Lesson 3
                                                        The Number i
   OBJECTIVES:
   At the end of this lesson, students are expected to:

           •recognize the property of the number i;
           •discuss the powers of i; and
           •solve the high powers of imaginary unit.


 Consider Equations 1 and 2 again.

                                        Equation 1               Equation 2
                                        x2 - 1 = 0.              x2 + 1 = 0.
                                        x2 = 1.                  x2 = -1.
         Equation 1 has solutions because the number 1 has two square roots, 1 and -1. Equation 2 has no solutions
 because -1 does not have a square root. In other words, there is no number such that if we multiply it by itself we get -
 1. If Equation 2 is to be given solutions, then we must create a square root of -1.

        The imaginary unit i is defined by




The definition of i tells us that i2 = -1. We can use this fact to find other powers of i.



                                                                                    contents     back       next
Example

      i3 = i2 * i = -1*i = -i.
      i4 = i2 * i2 = (-1) * (-1) = 1.

      Exercise:

      Simplify i8 and i11.

       We treat i like other numbers in that we can multiply it by numbers, we can add it to other numbers, etc. The
difference is that many of these quantities cannot be simplified to a pure real number.

     For example, 3i just means 3 times i, but we cannot rewrite this product in a simpler form, because it is not a real
number. The quantity 5 + 3i also cannot be simplified to a real number.

      However, (-i)2 can be simplified. (-i)2 = (-1*i)2 = (-1)2 * i2 = 1 * (-1) = -1.

      Because i2 and (-i)2 are both equal to -1, they are both solutions for Equation 2 above.




                                                                                        contents   back       next
Name: ___________________                            Section: _______
Instructor: ________________                         Date: _______                  Rating: ______




 Instruction: Express each number in terms of i and simplify.

 1.
               ______________________________________________________


 2.
               ______________________________________________________


 3.
               ______________________________________________________


 4.
               ______________________________________________________


 5.
               ______________________________________________________


                                                                  contents   back      next
6.
      ______________________________________________________


7.
      ______________________________________________________


8.
      ______________________________________________________


9.
      ______________________________________________________


10.
      ______________________________________________________


11.
      ______________________________________________________


12.
      ______________________________________________________


13.
      ______________________________________________________

                                                contents       back   next
Lesson 4
                                             The Complex Plane
OBJECTIVES:
At the end of this lesson, students are expected to:

      •distinguish the points on the plane;
      •differentiate the real and imaginary part; and
      •draw from memory the figure form by the plot points on the complex plane.


      A complex number is one of the form a + bi, where a and b are real numbers. a is called the real part of the
complex number, and b is called the imaginary part.

       Two complex numbers are equal if and only if their real parts are equal and their imaginary parts are equal. I.e.,
a+bi = c+di if and only if a = c, and b = d.

      Example.

      2 - 5i.
      6 + 4i.
      0 + 2i = 2i.
      4 + 0i = 4.

      The last example above illustrates the fact that every real number is a complex number (with imaginary part 0).
Another example: the real number -3.87 is equal to the complex number -3.87 + 0i.

       It is often useful to think of real numbers as points on a number line. For example, you can define the order
relation c < d, where c and d are real numbers, by saying that it means c is to the left of d on the number line.



                                                                              contents           back        next
We can visualize complex numbers by associating them with points in the plane. We do this by letting the
number a + bi correspond to the point (a,b), we use x for a and y for b.




 Exercises: Represent each of the following complex number by a point in the plane.

       1. 3 + 2i
       2. 1 – 4i
       3. 4 + 3i
       4. 2 – 5i
       5. 4 – 3i




                                                                            contents     back      next
Name: ___________________                           Section: _______
Instructor: ________________                        Date: _______                              Rating: ______




Instruction: Represent each of the following Complex Numbers by a point in the plane.

1.
             ______________________________________________________

2.           ______________________________________________________


3.           ______________________________________________________


4.   0       ______________________________________________________


5.   3       ______________________________________________________


6.           ______________________________________________________


7.   1/2     ______________________________________________________


8.           ______________________________________________________

                                                                   contents             back      next
9.
      _____________________________________________________



10.   ____________________________________________________



11.
      _____________________________________________________



12.   _____________________________________________________



13.   _____________________________________________________



14.
      _____________________________________________________


15.   _____________________________________________________




                                                 contents     back   next
Lesson 5
                                                    Complex Arithmetic
  OBJECTIVES:
  At the end of this lesson, students are expected to:

          •define the four arithmetic operations on complex numbers;
          •comply with the steps in solving the different operations; and
          •solve the four arithmetic operations.



       When a number system is extended the arithmetic operations must be defined for the new numbers, and the
important properties of the operations should still hold. For example, addition of whole numbers is commutative. This
means that we can change the order in which two whole numbers are added and the sum is the same: 3 + 5 = 8 and 5
+ 3 = 8.

       We need to define the four arithmetic operations on complex numbers.

Addition and Subtraction
       To add or subtract two complex numbers, you add or subtract the real parts and the imaginary parts.

                                                 (a + bi) + (c + di) = (a + c) + (b + d)i.
                                                  (a + bi) - (c + di) = (a - c) + (b - d)i.

 Example

 (3 - 5i) + (6 + 7i) = (3 + 6) + (-5 + 7)i = 9 + 2i.
 (3 - 5i) - (6 + 7i) = (3 - 6) + (-5 - 7)i = -3 - 12i.
                                                                                              contents   back   next
Note

      These operations are the same as combining similar terms in expressions that have a variable. For example, if
we were to simplify the expression (3 - 5x) + (6 + 7x) by combining similar terms, then the constants 3 and 6 would be
combined, and the terms -5x and 7x would be combined to yield 9 + 2x.

      The Complex Arithmetic applet below demonstrates complex addition in the plane. You can also select the other
arithmetic operations from the pull down list. The applet displays two complex numbers U and V, and shows their sum.
You can drag either U or V to see the result of adding other complex numbers. As with other graphs in these pages,
dragging a point other than U or V changes the viewing rectangle.

      Multiplication
      The formula for multiplying two complex numbers is

      (a + bi) * (c + di) = (ac - bd) + (ad + bc)i.

       You do not have to memorize this formula, because you can arrive at the same result by treating the complex
numbers like expressions with a variable, multiply them as usual, then simplify. The only difference is that powers of i do
simplify, while powers of x do not.

      Example

                           (2 + 3i)(4 + 7i)                   = 2*4 + 2*7i + 4*3i + 3*7*i2
                                                              = 8 + 14i + 12i + 21*(-1)
                                                              = (8 - 21) + (14 + 12)i
                                                              = -13 + 26i.




                                                                               contents            back       next
Notice that in the second line of the example, the i2 has been replaced by -1.

      Using the formula for multiplication, we would have gone directly to the third line.

      Exercise

      Perform the following operations.

      (a) (-3 + 4i) + (2 - 5i)
      (b) 3i - (2 - 4i)
      (c) (2 - 7i)(3 + 4i)
      (d) (1 + i)(2 - 3i)

      Division
      The conjugate (or complex conjugate) of the complex number a + bi is a - bi.

      Conjugates are important because of the fact that a complex number times its conjugate is real; i.e., its
imaginary part is zero.

      (a + bi)(a - bi) = (a2 + b2) + 0i = a2 + b2.

      Example


                           Number                    Conjugate          Product
                           2 + 3i                    2 - 3i             4 + 9 = 13
                           3 - 5i                    3 + 5i             9 + 25 = 34
                           4i                        -4i                16




                                                                               contents      back   next
Suppose we want to do the division problem (3 + 2i)   (2 + 5i). First, we want to rewrite this as a fractional
expression        .

       Even though we have not defined division, it must satisfy the properties of ordinary division. So, a number
divided by itself will be 1, where 1 is the multiplicative identity; i.e., 1 times any number is that number.

              So, when we multiply         by,        , we are multiplying by 1 and the number is not changed.

      Notice that the quotient on the right consists of the conjugate of the denominator over itself. This choice was
made so that when we multiply the two denominators, the result is a real number. Here is the complete division
problem, with the result written in standard form.




                                                                            contents             back         next
Exercise:
Write (2 - i)   (3 + 2i) in standard form.

We began this section by claiming that we were defining complex numbers so that some equations would have solutions.
So far we have shown only one equation that has no real solutions but two complex solutions. In the next section we will see
that complex numbers provide solutions for many equations. In fact, all polynomial equations have solutions in the set of
complex numbers. This is an important fact that is used in many mathematical applications. Unfortunately, most of these
applications are beyond the scope of this course. See your text (p. 195) for a discussion of the use of complex numbers in
fractal geometry.




                                                                             contents           back       next
Name: ___________________                             Section: _______
Instructor: ________________                          Date: _______                        Rating: ______




 Instruction: Perform the indicated operations and express the result in the form   .

 1.
                        _____________________________________________________

 2.
                        _____________________________________________________

 3.
                        _____________________________________________________

 4.
                        _____________________________________________________

 5.
                        _____________________________________________________

 6.                     _____________________________________________________


 7.                     _____________________________________________________



                                                                     contents       back     next
8.
      _____________________________________________________

9.
      _____________________________________________________

10.
      _____________________________________________________

11.
      _____________________________________________________

12.
      _____________________________________________________

13.
      _____________________________________________________


14.   _____________________________________________________


15.   _____________________________________________________




                                         contents       back   next
A. Define and/or describe each of the following terms.
       1. Imaginary part
       2. Real number
       3. Complex number
       4. Complex plane
       5. Imaginary unit
       6. Commutative property
       7. Complex conjugate
B. 1. Simplify:
       1. i15
       2. i25
       3. i106
       4. i207
       5. i21
   2. Perform the indicated operation and express each answer.
               a.       +

             b.       +

             c.           +

             d.           +

             e.           +

             f.           +
                                                                 contents   back   next
3. Represent each complex numbers by a point in the plane.

       a.     3–i
       b.     -2 + 4i
       c.     -3 + 3i
       d.     4 + 5i
       e.     -3 + 5i
4. Give the real part and the imaginary part of each complex numbers in #3.

5. Perform the indicated operations.

      a. (3 – 2i) + (-7 + 3i)
      b. (-4 + 7i) + (9 – 2i)
      c. (14 – 9i) + (7 – 6i)
      d. (5 + i) – (3 + 2i)
      e. (7 – 2i) – (4 – 6i)
      f. (8 + 3i) – (-4 – 2i)
      g. (3 – 2i) (3 +2i)
      h. (5 + 3i) (4 – i)
      i. (11 + 2i)2 (5 – 2i)
      j. (5 + 4i) / (3 – 2i)
      k. (4 + i) (3 – 5i) / (2 – 3i)
      l. (7 + 3i) / (3 – 3i / 4)




                                                                         contents   back   next
Chapter III




        In this chapter, the different ways of solving the quadratic equation are recalled. There are by using the
factoring, completing the square, by quadratic formula and solving by graphing. Students are given guides to
determine the most appropriate method to use.

 TARGET SKILLS:
       At the end of this chapter, students are expected to:

              • distinguish appropriate method in solving quadratic equation;
              • discuss and follow the steps in such different method; and
              • resolve quadratic equation using any method you want.


                                                                                contents     back       next
Lesson 6
                                                 Solving by factoring
    OBJECTIVES:
    At the end of this lesson, students are expected to:

          •define what is factoring;
          •discuss the Zero Factor Principle; and
          •solve equation by using the factoring method.


        Factoring – rearrange the equation; factor the left member; equate each factor to zero to
                 obtain the two roots.

•       Solve (x – 3)(x – 4) = 0.

       The Zero Factor Principle tells me that at least one of the factors must be equal to zero. Since at least one of
the factors must be zero, I'll set them each equal to zero:

        x – 3 = 0 or x – 4 = 0                  x = 3 or x = 4

        Solve: x = 3, 4

       Note that "x = 3, 4" means the same thing as "x = 3 or x = 4"; the only difference is the formatting. The "x = 3,
4" format is more-typically used.

        Checking x = 3 in (x – 3)(x – 4) = 0:

        ([3] – 3)([3] – 4) ?=? 0
    (3 – 3)(3 – 4) ?=? 0
           (0)(–1) ?=? 0
                 0 = 0

                                                                               contents             back       next
Checking x = 4 in (x – 3)(x – 4) = 0:
                ([4] – 3)([4] – 4) ?=? 0
                   (4 – 3)(4 – 4) ?=? 0
                            (1)(0) ?=? 0
                                 0 = 0
•   Solve x2 + 5x + 6 = 0.

    This equation is already in the form "(quadratic) equals (zero)" but, unlike the previous example, this
isn't yet factored. The quadratic must first be factored, because it is only when you MULTIPLY and get zero
that you can say anything about the factors and solutions. You can't conclude anything about the individual
terms of the unfactored quadratic (like the 5x or the 6), because you can add lots of stuff that totals zero.

    So the first thing I have to do is factor:

               x2 + 5x + 6 = (x + 2)(x + 3)

    Set this equal to zero:

               (x + 2)(x + 3) = 0

    Solve each factor:

              x + 2 = 0 or x + 3 = 0
              x = –2 or x = – 3
    The solution to x2 + 5x + 6 = 0 is x = –3, –2

    Checking x = –3 and x = –2 in x2 + 5x + 6 = 0:

               [–3]2 + 5[–3] + 6 ?=? 0
                9 – 15 + 6 ?=? 0
                9 + 6 – 15 ?=? 0



                                                                              contents            back          next
15 – 15 ?=? 0
                        0 = 0
       [–2] 2 + 5[–2] + 6 ?=? 0

               4 – 10 + 6 ?=? 0
               4 + 6 – 10 ?=? 0
                  10 – 10 ?=? 0
                        0 = 0

       So both solutions "check".

•      Solve x2 – 3 = 2x.

        This equation is not in "(quadratic) equals (zero)" form, so I can't try to solve it yet. The first thing I need to do
is get all the terms over on one side, with zero on the other side. Only then can I factor and solve:

        x2 – 3 = 2x
x2 – 2x – 3 = 0
(x – 3)(x + 1) = 0
x – 3 = 0 or x + 1 = 0
x = 3 or x = –1

       Then the solution to x2 – 3 = 2x is x = –1, 3

•      Solve (x + 2)(x + 3) = 12.

       The (10 + 2)(9 + 3) does not equal 12, you should never forget that you must have "(quadratic) equals (zero)"
before you can solve.
       So, tempting though it may be, the factors above equal to the other side of the equation and "solve". Instead,
multiply out and simplify the left-hand side, then subtract the 12 over to the left-hand side, and re-factor.




                                                                                    contents             back        next
(x + 2)(x + 3) = 12
x2 + 5x + 6 = 12
x2 + 5x – 6 = 0

(x + 6)(x – 1) = 0
x + 6 = 0 or x – 1 = 0
x = –6 or x = 1

       Then the solution to (x + 2)(x + 3) = 12 is x = –6, 1

•      Solve x(x + 5) = 0.

       To "solve" the equation for "x + 5 = 0", divide it by x. But it can't divide by zero; dividing off the x makes the implicit
assumption that x is not zero. Used the variable factors having variables and numbers (like the other factor, x + 5), a
factor can contain only a variable, so "x" is a perfectly valid factor. So set the factors equal to zero, and solve:

      x(x + 5) = 0
x = 0 or x + 5 = 0
x = 0 or x = –5

       Then the solution to x(x + 5) = 0 is x = 0, –5

•      Solve x2 – 5x = 0.
       Factor the x out of both terms, taking the x out front.

      x(x – 5) = 0
x = 0 or x – 5 = 0
x = 0 or x = 5

       Then the solution to x2 – 5x = 0 is x = 0, 5

       There is one other case of two-term quadratics that you can factor:



                                                                                   contents             back        next
•Solve x2 – 4 = 0.

               This equation is in "(quadratic) equals (zero)" form, it's ready to solve. The quadratic itself is a
difference of squares, then apply the difference-of-squares formula:

x2 – 4 = 0
(x – 2)(x + 2) = 0
x – 2 = 0 or x + 2 = 0
x = 2 or x = –2
Then the solution is x = –2, 2

Note: This solution may also be formatted as "x =      2“

Exercises: Solve:
       1.(x – 3)(x – 5) = 0.
       2.x2 + 6x + 7 = 0.
       3.x2 – 4 = 2x.
       4.x2 – 6x = 0.
       5.x2 – 8 = 0.




                                                                                 contents             back        next
Name: ___________________                            Section: _______
Instructor: ________________                         Date: ________                 Rating: _______




Instruction: Solve the following Quadratic Equation by Factoring Method.


1. x2 – 36 = 0
                 _____________________________________________________

2. x2= 25
                 _____________________________________________________

3. x2 – 12x + 35 = 0
              _____________________________________________________

4. x2 – 3x – 40 = 0
              _____________________________________________________

5. 2x2 – 5x = 3
              _____________________________________________________

6. 3x2 + 25x = 18
              _____________________________________________________

7. 15x2 – 2x – 8 = 0
              _____________________________________________________



                                                                  contents   back     next
8. 3x2 – x = 10
              _____________________________________________________

9. x2 + 6x – 27 = 0
              _____________________________________________________

10. y2 – 2y – 3 = y – 3
               _____________________________________________________

11. 4y2 + 4y = 3
              _____________________________________________________

12. 3a2 + 10a = -3
             _____________________________________________________

13. a2 – 2a – 15 = 0
              _____________________________________________________

14. r2 + 6r – 27 = 0
               _____________________________________________________
15. 2z2 – 2 – 1 = 0
              _____________________________________________________




                                                        contents       back   next
Lesson 7
                            Solving by Completing the Square
    OBJECTIVES:

    At the end of this lesson, students are expected to:

           •analyze the techniques in completing the square;
           •comply with the techniques of completing the square; and
           •carefully change the exact signs for every equation.



     Some quadratics is fairly simple to solve because they are of the form "something-with-x squared equals some
number", and then you take the square root of both sides. An example would be:

       (x – 4)2 = 5
x – 4 = sqrt(5)
x = 4 sqrt(5)
x = 4 – sqrt(5) and x = 4 + sqrt(5)

        Unfortunately, most quadratics doesn’t come neatly squared like this. For your average everyday quadratic, you
first have to use the technique of "completing the square" to rearrange the quadratic into the neat "(squared part) equals
(a number)" format demonstrated above. For example:

•      Find the x-intercepts of y = 4x2 – 2x – 5.

       First off, remember that finding the x-intercepts means setting y equal to zero and solving for the x-values, so this
question is really asking you to "Solve 4x2 – 2x – 5 = 0".



                                                                               contents            back        next
The answer can also be written in rounded form as

                                                    contents   back   next
You will need rounded form for "real life" answers to word problems, and for graphing. But (warning!) in
most other cases, you should assume that the answer should be in "exact" form, complete with all the square
roots.

        When you complete the square, make sure that you are careful with the sign on the x-term when you
multiply by one-half. If you lose that sign, you can get the wrong answer in the end, because you'll forget what
goes inside the parentheses. Also, don't be sloppy and wait to do the plus/minus sign until the very end. On your
tests, you won't have the answers in the back, and you will likely forget to put the plus/minus into the answer.
Besides, there's no reason to go ticking off your instructor by doing something wrong when it's so simple to do it
right. On the same note, make sure you draw in the square root sign, as necessary, when you square root both
sides. Don't wait until the answer in the back of the book "reminds" you that you "meant" to put the square root
symbol in there. If you get in the habit of being sloppy, you'll only hurt yourself!

•     Solve x2 + 6x – 7 = 0 by completing the square.

       Do the same procedure as above, in exactly the same order. (Study tip: Always working these problems in
exactly the same way will help you remember the steps when you're taking your tests.)




                                                                           contents           back        next
If you are not consistent with remembering to put your plus/minus in as soon as you square-root both
sides, then this is an example of the type of exercise where you'll get yourself in trouble. You'll write your answer
as "x = –3 + 4 = 1", and have no idea how they got "x = –7", because you won't have a square root symbol
"reminding" you that you "meant" to put the plus/minus in. That is, if you're sloppy, these easier problems will
embarrass you!

      Exercise:

             1.   3x2 – 4x – 6 = 0
             2.   2x2 -3x + 4 = 0
             3.   x2 – 8x + 16 = 0
             4.   x2 + 18x + 72 = 0
             5.   2x2 – 6x + 1 = 0
                                                                            contents            back        next
Name: ___________________                           Section: _______
Instructor: ________________                        Date: _______                      Rating: ____




Instruction: Solve the following Quadratic Equation by Completing the Square.

1. x2 + 3x = 4
                 _____________________________________________________

2. x2 – 2x = 24
              _____________________________________________________

3. x2 + 4 = 4x
                 _____________________________________________________

4. 2x2 – 6 = x
                 _____________________________________________________

5. 4a2 + 12a + 9 = 0
              _____________________________________________________

6. 3a2 – 5 = 14a
              _____________________________________________________

7. 16b2 + 1 = 16b
              _____________________________________________________



                                                                   contents     back      next
8. 9b2 – 6b – 1 = 0
              _____________________________________________________

9. 9z2 + 30z + 20 = 0
              _____________________________________________________

10. 2a2 + a = 10a
              _____________________________________________________

11. 2x2 + 17 = 10x
              _____________________________________________________

12. 2a2 + 6a + 9 = 0
              _____________________________________________________

13. 5x2 – 2x + 1 = 0
              _____________________________________________________

14. 3x2 + 2x + 1= 0
              _____________________________________________________

15. 2y2 + 5y = 42
              _____________________________________________________




                                                         contents     back   next
Lesson 8
                                             Quadratic Formula
   OBJECTIVES:
   At the end of this lesson, students are expected to:

          •follow the step in solving quadratic formula;
          •distinguish the roots of the quadratic equation; and
          •perform substituting the values in the quadratic formula.


The following steps will serve as guide in solving this method.

Step 1. First subtract c from both sides of the equation and then, divide both sides by
(a # 0 by hypothesis) to obtain the equivalent equation,

                                             x2 +     =

Step 2. Complete the left-hand side in to the perfect square.

             x2 + bx/a + (b/2a)2 = (b/2a)2 – c/a
             or (x+b/2a)2 = (b2-4ac)/4a2

Step 3. Take the square roots of both sides of the last equation.

             (x+b/2a) =    (           /2a




                                                                              contents    back   next
Step 4. Solve for x.

              x ==             or

Let a, b and c be real constant, where a ≠ 0. Then the roots of ax2 + bx + c = 0 are
                 =
              x=


              The above formula is referred to as the quadratic formula.

Example: Solve               a. 3x2 – x – 5/2 = 0

                   Solutions: Here a=3, b=⁻1, c=⁻5/2

Substituting these values in the quadratic formula

          we obtain x =


                       =

                       =

         The roots are              and      .

            a. 2x2 – 5 (x-2) = 8
To be able to apply the formula, we must first put the given equation in standard form.

       2x2 – 5 (x-2) = 8
       2x2 – 5x + 10 = 8
       2x2 – 5x + 2 = 0

                                                                             contents     back   next
Here a=2, b=⁻5 c=2. By the quadratic formula

.                 x=                        =


    The roots are 2 and        .
                                   Note that the expression 2x2 – 5x + 2 can be factored as
                                                 2x2 – 5x + 2 = (2x – 1) (x – 2)

                                   The roots of the quadratic equation x = 1/2 and x = 2. This example
    shown that if we can see that the given equation in factorable, it will be quicker to solve it by factoring.

    Exercises: Solve each equation by quadratic formula.
                         1.   x2 – 14x + 49 = 0
                         2.   x2 – 4x – 21 = 0
                         3.   x2 + 5x – 36 = 0
                         4.   x2 + x – 30 = 0
                         5.   x2 + 3x = 40




                                                                                     contents            back      next
Name: ___________________                            Section: _______
Instructor: ________________                         Date: ________                      Rating: ______




Instruction: Solve the following equations by the Quadratic Formula.

1. 2a2 – 10 = 9
                    _____________________________________________________

2. 6b2 – b = 12
                    _____________________________________________________

3. 3x2 + x = 14
                    _____________________________________________________

4. 10a2 + 3 = 11a
                    _____________________________________________________

5. 2x2 + 5x = 12
                    _____________________________________________________

6. 4x2 + 5x = 21
                    _____________________________________________________




                                                                       contents   back      next
7. 2x2 – 7x + 3 = 0
              _____________________________________________________

8. 3a2 – 6a + 2 = 0
              _____________________________________________________

9. 3b2 – 2b – 4 = 0
              _____________________________________________________

10 a2 – 3a – 40 = 0
              _____________________________________________________

11. 3y2 – 11y + 10 = 0
              _____________________________________________________

12. 3w2 = 9 + 2w
             _____________________________________________________

13. 15z2 + 22z = 48
             _____________________________________________________

14. 9a2 + 14 = 24a
              _____________________________________________________

15. 16m2 = 24m + 19
             _____________________________________________________




                                                        contents      back   next
Lesson 9
                                               Solving "by Graphing
  OBJECTIVES:

  At the end of this lesson, students are expected to:

          •define graphing;
          •resolve the equation by graphing; and
          •draw the points from the equations given.



       To be honest, solving "by graphing" is an achingly trendy but somewhat bogus topic. The basic idea behind
solving by graphing is that, since the "solutions" to "ax2 + bx + c = 0" are the x-intercepts of "y = ax2 + bx + c", you
can look at the x-intercepts of the graph to find the solutions to the equation. There are difficulties with "solving"
this way, though....
       When you graph a straight line like "y = 2x + 3", you can find the x-intercept (to a certain degree of
accuracy) by drawing a really neat axis system, plotting a couple points, grabbing your ruler and drawing a nice
straight line, and reading the (approximate) answer from the graph with a fair degree of confidence.On the other
hand, a quadratic graphs as a wiggly parabola. If you plot a few non-x-intercept points and then draw a curvy line
through them, how do you know if you got the x-intercepts even close to being correct? You don't. The only way
you can be sure of your x-intercepts is to set the quadratic equal to zero and solve. But the whole point of this
topic is that they don't want you to do the (exact) algebraic solving; they want you to guess from the pretty
pictures.
        So "solving by graphing" tends to be neither "solving" nor "graphing". That is, you don't actually graph anything, and
you don't actually do any of the "solving". Instead, you are told to punch some buttons on your graphing calculator and look
at the pretty picture, and then you're told which other buttons to hit so the software can compute the




                                                                                  contents             back         next
intercepts (or you're told to guess from the pretty picture in the book, hoping that the printer lined up the
different print runs for the different ink colors exactly right). I think the educators are trying to "help" you "discover"
the connection between x-intercepts and solutions, but the concept tends to get lost in all the button-pushing.
Okay, enough of my ranting...
    To "solve" by graphing, the book may give you a very neat graph, probably with at least a few points labeled; the
book will ask you to state the points on the graph that represent solutions. Otherwise, it will give you a quadratic,
and you will be using your graphing calculator to find the answer. Since different calculator models have different
key-sequences, I cannot give instruction on how to "use technology" to find the answers, so I will only give a couple
examples of how to solve from a picture that is given to you.

•     Solve x2 – 8x + 15 = 0 by using the following graph.




            The graph is of the related quadratic, y = x2 – 8x + 15, with the x-intercepts being where y = 0. The point here
    is to look at the picture (hoping that the points really do cross at whole numbers, as it appears), and read the x-
    intercepts (and hence the solutions) from the picture.

           The solution is x = 3,

           Since x2 – 8x + 15 factors as (x – 3)(x – 5), we know that our answer is correct.

                                                                                contents           back        next
• Solve 0.3x2 – 0.5x – 5/3 = 0 by using the following graph.




        For this picture, they labeled a bunch of points. Partly, this was to be helpful, because the x-intercepts are
 messy (so I could not have guessed their values without the labels), but mostly this was in hopes of confusing me,
 in case I had forgotten that only the x-intercepts, not the vertices or y-intercepts, correspond to "solutions".

       The x-values of the two points where the graph crosses the x-axis are the solutions to the equation.

       The solution is x = –5/3,   10/
                                         3

 • Find the solutions to the following quadratic:




                                                                           contents            back       next
They haven't given me the quadratic equation, so I can't check my work algebraically. (And, technically, they
haven't even given me a quadratic to solve; they have only given me the picture of a parabola from which I am
supposed to approximate the x-intercepts, which really is a different question....)

      I ignore the vertex and the y-intercept, and pay attention only to the x-intercepts. The "solutions" are the x-
values of the points where the pictured line crosses the x-axis:

       The solution is x = –5.39, 2.76

       "Solving" quadratics by graphing is silly in "real life", and requires that the solutions be the simple factoring-
type solutions such as "x = 3", rather than something like "x = –4 + sqrt(7)". In other words, they either have to
"give" you the answers (by labeling the graph), or they have to ask you for solutions that you could have found
easily by factoring. About the only thing you can gain from this topic is reinforcing your understanding of the
connection between solutions and x-intercepts: the solutions to "(some polynomial) equals (zero)" correspond to
the x-intercepts of "y equals (that same polynomial)". If you come away with an understanding of that concept, then
you will know when best to use your graphing calculator or other graphing software to help you solve general
polynomials; namely, when they aren't factorable.




                                                                             contents            back        next
Name: ___________________                       Section: _______
Instructor: ________________                    Date: _______                  Rating: ______




Instruction: Solve each equation by graphing.

1. x2 – 6x + 9 = 0
                _____________________________________________________

2. x2 – 5x + 10 = 0
               _____________________________________________________

3. 2x2 – 6x + 8 = 0
               _____________________________________________________

4. x2 – 7x + 12 = 0
               _____________________________________________________

5. 2x2 – 8x + 10 = 0
              _____________________________________________________

6. 3x2 + 6x – 9 = 0
              _____________________________________________________

7. x2+ 8x – 12 = 0
              _____________________________________________________


                                                           contents     back    next
8. 2 + 4x – 3 = 0
                _____________________________________________________

9. x2 – 2x – 2 = 0
                _____________________________________________________

10. 2x2 – 4x – 2 = 0
               _____________________________________________________

11. 4x2 – 8x – 16 = 0
               _____________________________________________________

12. x2 – 9x + 21 = 0
               _____________________________________________________

13. x2 + 10x + 18 = 0
               _____________________________________________________

14. 2x2 – 16x + 8 = 0
               _____________________________________________________

15. 3x2 – 12x – 9 = 0
               _____________________________________________________




                                                          contents      back   next
A. Solve by factoring.
       1. x2 – 3x – 10 = 0
       2. x2 + 2x = 8
       3. x2 – x – 4 = 2
       4. 2x2 – 6x – 36 = x2 – 15
       5. 4x2 + 4x = 15
       6. 6x2 + 11x – 2 = 8
       7. 49x2 + 28x – 10 = 0
       8. 6x4 – 4x3 – 10x2 = 0
       9. 18 + 15x – 18x2 = 0
       10. x4 – 4x2 + 3 = 0

B. Solve by completing the square.
       1. x2 - 4x – 3 = 0
       2. x2 + 3x – 6 = 0
       3. x2 – 7x + 5 = 0
       4. 2x2 + 5x + 1 = 0
       5. 2x2 + 8x – 5 = 0

C. Solve for x by the quadratic formula.
       1. x2.- 4x – 7 = 0
       2. x2 – 3x + 4 = 0
       3. 2x2 + 4x + 5 = 0
       4. x2 + 7x – 3 = 0
       5. x2 – 7x + 2 = 0
       6. x2 + 5x – 7 = 0
       7. x2 + 9x – 3 = 0
       8. 4x2 – 6x + 2 = 0
       9. 9x2 – 9x – 10 = 0
       10. x2 + 5x + 8 = 0                 contents   back   next
Chapter IV




        Much of the study in quadratic equation consist of different solving equation, we have equation in
 quadratic form, equation containing radicals and equation reducible to quadratic equation. They have their own
 steps and procedures to be followed in order to solve the given equation.


   TARGET SKILLS:

         At the end of this chapter, students are expected to:
                • discuss solving equation on quadratic;
                • determine the index and its radicals;
                • interpret the solution of the original equation; and
                • select appropriate method in solving quadratic equation.




                                                                        contents          back       next
Lesson 10
                                       Equation in Quadratic Form
 OBJECTIVES:

 At the end of this lesson, students are expected to:

        •identify equation in quadratic form;
        •select appropriate method in solving quadratic equation; and
        •change the equation in standard form.



Quadratic in Form

      An equation is quadratic in form when it can be written in this standard form

where the same expression is inside both ( )'s.

      In other words, if you have a times the square of the expression following b plus b times that same
expression not squared plus c equal to 0, you have an equation that is quadratic in form.
      If we substitute what is in the ( ) with a variable like t, then the original equation will become a quadratic
equation.




                                                                          contents            back       next
Solving Equations that are
                                    Quadratic in Form
Step 1: Write in Standard Form,                            , if needed.

         If it is not in standard form, move any term(s) to the appropriate side by using the
         addition/subtraction property of equality.

         Also, make sure that the squared term is written first left to right, the expression not squared is
         second and the constant is third and it is set equal to 0.

Step 2: Substitute a variable in for the expression that follows b in the second term.

         In other words, substitute your variable for what is in the ( ) when it is in standard form, .



         I’m going to use t for my substitution, but really you can use any variable as long as it is not the
         variable that is used in the original equation.

Step 3: Solve the quadratic equation created in step 2.

         You can use any method you want to solve the quadratic equation: factoring, completing the square or
         quadratic formula.
Step 4: Find the value of the variable from the original equation.
         Keep in mind that you are finding a solution to the original equation and that the variable you
         substituted in for in step 2 is not your original variable.

         Use the substitution that was used to set up step 2 and then solve for the original variable.

                                                                           contents             back      next
.
    Step 5: Check your solutions.
            In some cases, you will be working with rational exponents and square roots in your problems. Those
            types of equations can cause extraneous solutions. Recall that an extraneous solution is one that is a
            solution to an equation after doing something like raising both sides of an equation by an even power,
            but is not a solution to the original problem.

            Even though not all of the quadratic in form equations can cause extraneous solutions, it is better to be
            safe than sorry and just check them all.

     Example 1: Solve the equation that is quadratic in form:

                Standard Form,

                                                             *Rewriting original equation to show it is quadratic in form
                                                             *Note that (y squared) squared = y to the fourth

                                                             *When in stand. form, let t = the expression following b.

                Next, we need to substitute t in for y squared in the original equation.


                                                             *Original equation




                                                             *Substitute t in for y squared


                                                                                  contents            back        next
Note how we ended up with a quadratic equation when we did our substitution. From here, we need
to solve the quadratic equation that we have created.


Solve the quadratic equation: factoring, completing the square or quadratic formula.




                                            *Factor the trinomial



                                            *Use Zero-Product Principle
                                            *Set 1st factor = 0 and solve




                                            *Set 2nd factor = 0 and solve




                                                                contents           back   next
Let's find the value(s) of y when t = -4:

                                            *Plug in - 4 for t
                                            *Use square root method to solve for y

                                            *First solution




                                            *Second solution


 Let's find the value(s) of y when t = 1:



                                             *Plug in 1 for t
                                             *Use square root method to solve for y

                                             *First solution




                                             *Second solution
                                                                  contents            back   next
.
    Example 2: Solve the equation that is quadratic in form:

                Standard Form,



                                                         *Inverse of add. 3 is sub. 3

                                                         *Equation in standard form



                Note how when you square x to the 1/3 power you get x to the 2/3 power, which is what you have
                in the first term.

                                                         * Rewriting original equation to show it is quadratic in
                                                         form
                                                         *Note that (x to the 1/3 power) squared = x to the 2/3
                                                         power

                                                         *When in stand. form, let t = the expression following b.




                 Next, we need to substitute t in for x to the 1/3 power in the original equation.




                                                                               contents              back      next
*Original equation




                                      *Substitute t in for x to the 1/3 power


You can use any method you want to solve the quadratic equation: factoring, completing the square
or quadratic formula.


                                      *Factor the trinomial



                                      *Use Zero-Product Principle
                                      *Set 1st factor = 0 and solve




                                      *Set 2nd factor = 0 and solve




                                                           contents             back   next
Let's find the value(s) of x when t = 3:


                                            *Plug in 3 for t
                                            *Solve the rational exponent equation

                                            *Inverse of taking it to the 1/3 power is
                                            raising it to the 3rd power



Let's find the value(s) of x when t = -1:


                                            *Plug in -1 for t
                                            *Solve the rational exponent equation

                                            *Inverse of taking it to the 1/3 power is
                                            raising it to the 3rd power


Let's double check to see if x = 27 is a solution to the original equation.


                                            *Plugging in 27 for x



                                            *True statement



                                                                contents            back   next
Since we got a true statement, x = 27 is a solution.

                 Let's double check to see if x = -1 is a solution to the original equation.


                                                          *Plugging in -1 for x



                                                          *True statement


                 Since we got a true statement, x = -1 is a solution.

                 There are two solutions to this equation: x = 27 and x = -1.


Exercises:
       1.    a4 + 2a2 – 5 = 0
       2.    x2 – 3x + 2 = 0
       3.    s6 + 8s3 – 6 = 0
       4.    n2 – 6n + 10 = 0
       5.    g8 + 2g4 – g = 0




                                                                                  contents     back   next
Name: ___________________                              Section: _______
Instructor: ________________                           Date: _______                Rating: ______




Instruction: Solve the equation that is in quadratic form.


1. a8 + 2a4 – 8 = 0
                _____________________________________________________

2. l2 + 4l2 – 6 = 0
                 _____________________________________________________

3. e4 – 8e2 – 3 = 0
                _____________________________________________________

4. l6 – 10l – 5 = 0
                 _____________________________________________________

5. i10 – 8i5 – 4 = 0
                  _____________________________________________________

6. s6 – 5s3 – 25 = 0
                _____________________________________________________




                                                                  contents   back     next
7. h2/4 + 8h1/4 – 12 = 0
                 _____________________________________________________

8. a6- 5a4 – 15 = 0
               _____________________________________________________

9. n8 + 12n2 – 8 = 0
               _____________________________________________________

10. e9 – 3n3 – 10 = 0
               _____________________________________________________

11. x2/3 – 2x 1/3 = 8
                 _____________________________________________________

12. x3/6 – 3x1/2 = 9
                 _____________________________________________________

13. y2- 8y = 5
                 _____________________________________________________

14. y4 + 2y2 = 6
               _____________________________________________________

15. x6 – 9x2 + 8 = 0
                _____________________________________________________




                                                           contents      back   next
Lesson 11
                                     Equation Containing Radicals
  OBJECTIVES:

  At the end of this lesson, students are expected to:

            •determine the index and its radicals;
            •positively respond to the note to be remembered; and
            •perform isolation of one radical if there are two radicals in the equation.


               In the radicals     which is read the “nth root of b,” the positive integer n is called the index or
order of the radical, and b is called its radicand. When n is 2, 2 is no longer written, just simply write  instead
of      to indicate the square root of b, thus      is read as “cube root of b”;     as “4 th of b”.



Note:

        •       In order to solve for x, you must isolate x.
        •       In order to isolate x, you must remove it from under the radial.
        •       If there are two radicals in the equation, isolate one of the radicals.
        •       Then raise both sides of the equation to a power equal to the index of the isolated radical.
        •       Isolate the the remaining radical.
        •       Raise both sides of the equation to a power equal to the index of the isolated radical.
        •       You should now have a polynomial equation. Solve it.
        •       Remember that you did not start out with a polynomial; therefore, there may be extraneous solutions.
                Therefore, you must check your answers.




                                                                                 contents       back        next
Example 1:


              First make a note of the fact that you cannot take the square root of a negative number. Therefore,
the       term is valid only if       and the second term             is valid if

Isolate the           term

Square both sides of the equation.




Isolate the       term


Square both sides of the equation.




              Check the solution by substituting 9 in the original equation for x. If the left side of the equation
equals the right side of the equation after the substitution, you have found the correct answer.




                                                                            contents            back        next
• Left side:
•Right Side:1
•
             Since the left side of the original equation does not equal the right side of the original equation
after we substituted our solution for x, then there is no solution.

You can also check the answer by graphing the equation:




.The graph represents the right side of the original equation minus the left side of the original equation.. The x-
                      =5
intercept(s) of this graph is (are) the solution(s). Since there are no x-intercepts, there are no solutions.

Exercises:

Solve each of the following equation.

1.              =3

2.              =x+1

3.              =

4.              =5

5.              = 10




                                                                        contents            back       next
Name: ___________________                             Section: _______
Instructor: ________________                          Date: _______               Rating: ______




 Instruction: Solve each of the following equation.


 1.
                      _____________________________________________________

 2.
                      _____________________________________________________

 3.
                      _____________________________________________________

 4.
                      _____________________________________________________

 5.
                      _____________________________________________________

 6.
                      _____________________________________________________




                                                                    contents   back   next
7.    ____________________________________________________


8.    ____________________________________________________


9.    ____________________________________________________


10.
      ____________________________________________________


11.
      ____________________________________________________

12.
      ____________________________________________________

13.
      ____________________________________________________

14.
      ____________________________________________________

15.
      ____________________________________________________




                                         contents       back   next
Lesson 12
                         Equations Reducible to Quadratic Equations
  OBJECTIVES:
  At the end of this lesson, students are expected to:

         •interpret the solution of the original equation;
         •organize the equation if it is quadratic equation; and
         •solve the equation by factoring or quadratic formula.


       A variety of equations can be transformed into quadratic equations and solved by methods that we have
discussed in the previous section. We will consider fractional equations, equations involving radicals and
equation that can be transformed into quadratic equations by appropriate substitutions. Since the transformation
process may introduce extraneous roots which are not solutions of the original equation, we must always check
the solution in the original equation.

      Example: Solve     1     1    7
                       x+2 + x+3 = 12

     Solution: First note that neither -2 nor -3 can be a solution since at either of these points the equation is
meaningless.

      Multiplying by the LCD, 12(x+2) (x+3), we get

      12(x+3) + 12(x+2) = 7(x+2) (x+3)
      24x + 60 = 7(x2 + 5x + 6)
      or
      7x2 + 11x – 18 = 0

      Factoring, we get, (7x + 18)(x – 1) = 0

                                                                           contents           back        next
x = 1 or -18
          7

If x = 1, _1_ _1_ _1_ _1_ _7_
          1+2 1+3 = 3 + 4 = 12

Therefore x = 1 is a solution.

If x = -18, __1__ +   __1__
        7 -18/7 + 2 -18/7 + 3

= __7__ + __7__
-18 + 14 -18 + 21

= _-7_ + _7_ = _7_
   4      3    12

Therefore, x = _-18_ is a solution.
                 7

Example 2.               =            -2

Solution: squaring both sides of the equation, we obtain,


                     =           +4

         2x – 16 = 4




                                                            contents   back   next
Dividing both sides by 2 gives, x – 8 = -2

Squaring both sides of the equation we get
                                  2
 (x - 8)        =

 x2 - 16x + 64 = 4 (x +16)

 x2 – 20x = 0

 x(x – 20) =0

 x = 0 or x = 20

 Check: if x = 0,            =

       =

 8=6–2
 8≠4

 Therefore x = 20 is not a solution of the original equation.
 Thus the only root of

        Many equations are not quadratics equations. However, we can transform them by means of appropriate
 substitutions into quadratics equations and then solve these by techniques that we know.




                                                                       contents          back       next
Example: Solve:

       1. 2x-2 – 7x-1 + 3 = 0

       2. x4 – 2x2 – 2 = 0

       3.         2    –


Solutions

a.     Let u = x-1. Then u2 = (x-1)2 = x-2 and our equation becomes

2u2 – 7u + 3 = 0, a quadratic equation in u.

To solve the equation, we factor the left-hand side.

(2u – 1)( u – 3) = 0
U = ½ or u = 3
Since u = x-1, x-1=          or x-1 = 3, from which

 x = 2 or x =

Check: if x = 2, 2(2-2) – 7(2-1) + 3 =

Thus x = 2 is solution

If x = 1/3, 2(1/3)-2 – 7(1/3)-1 + 3 = 2(3)2 – 7(3) + 3

So, x = 1/3 is a solution.

          b. Let u = x2. Then u2 = x4 and the given equation becomes a quadratic equation in u.
u2 – 2u – 2 = 0
                                                                         contents            back   next
u=                      =


.   u=1+         or u =

    Since u = x2 and u =             < 0, we have to discard this solution.
.
    u = x2 =            implies

                   x=

                  It is simple to verify that both values of x satisfy the original equation. The roots of x4 – x2 – 2 = 0 are
               and -

                   c. Let u =        .This substitution yields a quadratic equation in u.
                                     u2 – u – 2 = 0
                                     (u – 2)(u + 1) = 0
                                     u = 2 0r u = ˉ1
                    u=          =2     implies x = 2(4x + 1)
                                                          or       x=

                    u=          =1     implies   x = ˉ4x – 1
                                                           or      x=


      Again, it can easily be verified that both solutions check in the original equation.

               The roots are         and



                                                                                  contents            back        next
Name: ___________________                      Section: _______
Instructor: ________________                   Date: _______         Rating: ______




 Instruction: Solve the following equation.

 1.
                        _____________________________________________________

 2.                     _____________________________________________________


 3.
                        _____________________________________________________

 4.
                        _____________________________________________________

 5.
                         _____________________________________________________

 6.
                         _____________________________________________________




                                                            contents       back       next
7.
      _____________________________________________________

8.
      _____________________________________________________

9.
      _____________________________________________________


10.   _____________________________________________________


11.
      _____________________________________________________

12.
      _____________________________________________________
13.
       _____________________________________________________
14.
       _____________________________________________________

15.
       _____________________________________________________




                                          contents       back   next
A. Solve for x.                    C. Solve for x.

                                   1.     +       =
1.          =4

2.             -5=0
                                   2.         +       +       =0
3.      =
                                   3.         +           =
4.            =

5.             =x+2
                                   4.         +       =2

B. Reduce to quadratic equation.
                                   5.         +       =
1. x4 – 5x + 4 = 0
2. 4(x + 3) + 5         = 21
3. x2/3 – 5x1/3 – 6 = 0
4. (x2 + 4x)2 – (x2 + 4x) = 20
5. 2x4 – 9x2 + 7 = 0




                                                              contents   back   next
Chapter V




         The discriminant gives additional information on the nature of the roots beyond simply whether there
 are any repeated roots: it also gives information on whether the roots are real or complex, and rational or
 irrational. More formally, it gives information on whether the roots are in the field over which the polynomial is
 defined, or are in an extension field, and hence whether the polynomial factors over the field of coefficients.
 This is most transparent and easily stated for quadratic and cubic polynomials; for polynomials of degree 4 or
 higher this is more difficult to state.

   TARGET SKILLS:

          At the end of this chapter, students are expected to:

                 •determine discriminant, roots and coefficient;
                 • discuss the relation the roots and coefficient;
                 • find the sum and product of the roots; and
                 • change quadratic equation to discriminant formula.


                                                                           contents            back        next
Lesson 13
                           The Discriminant and the roots of a
                                   Quadratic Equation
  OBJECTIVES:
  At the end of this lesson, students are expected to:

          •determine discriminant and the roots;
          •compare discriminant and the nature of the roots; and
          •change quadratic equation to discriminant using the nature of the roots.


Example

1. Find the x-intercept of y = 3x² - 6x + 4.

Solution: As already mentioned, the values of x for which 3x² - 6x + 4 = 0 give the x-intercepts of the function.
We apply the quadratic formula in solving the equation.

              3x² - 6x + 4 = 0
              x=                     =

             Since         is not a real number, the equation 3x² - 6x + 4 = 0 has no real root. This means that the
 parabola y = 3x² - 6x + 4 does not intersect the x-axis.

               Let us write the equation in the form y = a(x – h)²+ k.

                             y = 3(x² – 2x)² + 4
                               = 3(x – 1)² + 1


                                                                            contents           back        next
∆ = b² - 4ac                                      Roots of ax² + bx + c = 0
Positive                                                 Real and distinct

                                                          r=

                                                          s=


Zero                                                     Real and equal

                                                          r=s=


Negative                                                 No real roots

Example 2. Use the disciminant to determine the nature of the roots of the
           following quadratic equation.
         a. x² - x + ¼ = 0
         a = 1, b = ˉ1, c = ¼
         b² - 4ac = (ˉ1)² - 4 (1)(¼)
         =1–1
         =0                 There is only one solution, that is, a double root.
                            Note that x² - x = ½ = (x - ½), so that double root is ˉb/2a = ½.

           b. 5x² - 4x + 1 = 0
           a = 5, b = ˉ4, c = 1
           b² - 4ac = (ˉ4)² - 4 (5)(1)
           = 16 – 20
           = ˉ4 < 0           There are no real roots since a negative number has no real square root.


                                                                             contents           back     next
Name: ___________________                             Section: _______
Instructor: ________________                          Date: _______               Rating: ______




Instruction: Use the Discriminant to determine the nature of the root of the following Quadratic Equations.

1.x2- 2x – 3=0
                  _____________________________________________________

2. 6x2 – x – 1 = 0
                _____________________________________________________

3. 2x2 – 50 = 0
                  _____________________________________________________

4. x2 – 8x + 12 = 0
               _____________________________________________________

5. x2 + 5x – 14 = 0
                _____________________________________________________

6. -4x2 – 4x + 1 = 0
                _____________________________________________________

7. 7x2 + 2x – 1 = 0
                  _____________________________________________________




                                                                     contents           back        next
8.x2 + 3x = 40
                 _____________________________________________________

9. 3x2= 5x – 1
                 _____________________________________________________

10. 3x2+ 12 – 1=0
               _____________________________________________________

11. (x-2)(x-3) = 4
               _____________________________________________________

12.2x2 + 2x + 1 = 0
               _____________________________________________________

13. 7x2 + 3 – 6x = 0
               _____________________________________________________

14. 5x2 – 6x + 4 = 0
               _____________________________________________________

15. 3x2 + 2x + 2 = 0
               _____________________________________________________




                                                            contents     back   next
Lesson 14
                        Relation between roots and coefficient
  OBJECTIVES:
  At the end of this lesson, students are expected to:

         •classify roots and coefficient;
         •discuss relations between the roots and coefficient of the quadratic equation; and
         •find the sum and product of the roots of a given quadratic equation


              There are some interesting relations between the sum and the product of the roots of a quadratic
equation. To discover these, consider the quadratic equation ax2 + bx + c = 0, where a ≠ 0.

             Multiply both sides of this equation by 1/a so that the coefficient of x2 is 1.
                                        (ax2 + bx + c) =


We obtain an equivalent quadratic equation in the form

                                       x2 +     +     =0


       If r and s are the roots of the quadratic equation ax2 + bx + c = 0, then from the quadratic formula

                                r=               and s =




                                                                               contents         back          next
Adding the roots, we obtain

              r+s=                   +

                   =      =


 Multiplying the roots, we obtain

                rs =

                                   = c/a


            Observe the coefficient in the quadratic equation x2 + bx/a + c/a = 0. How do they compare with the
sum and the product of the roots? Did you observe the following?

              1. The sum of the roots is equal to the negative of the coefficient of x.
                                          r + s = -b / a

              2. The product of the roots is equal to the constant term
                                          rs = c / a

An alternate way of arriving at these relations is as follows

                              Let r and s be the roots of x2 + bx/a + c/a = 0. Then

                                            x - r)(x – s) = 0

Expanding gives, x2 – rx – sx + rs = 0

                              or x2 – (r + s)x + rs = 0
                                                                                contents     back       next
Comparing the coefficients of the corresponding terms, we obtain

                                               r + s = -b / a and rs c / a

             The above relations between the roots and the coefficients provide a fast and convenient means of
checking the solutions of a quadratic equation.

Example: Solve and check. 2x2 + x – 6 = 0

Solutions: 2x2 + x – 6 = (2x – 3)(x + 2) = 0

                                  x = 3/2 or x = 2

              The roots are 3/2 and 2.

To check, we add the roots, 3/2 = (-2) = -1/2 = -b/a.

and multiply them 3/2 = (-2) = -3 = c/a
Example: Find the sum and the product of the roots of 3x2 – 6x + 8 = 0 without having to first determine the roots.

Solution: The sum of the roots is r + s = -c/a = -(-6)/3 = 2

and their product is rs = c/a = 8/3




                                                                             contents          back        next
Name: ___________________                              Section: _______
Instructor: ________________                           Date: _______                              Rating: ______




Instruction: Without solving the roots, find the sum and product of the roots of the following.

1. 6x2 – 5x + 2 = 0
                      _____________________________________________________

2. x2 + x – 182 = 0
                      _____________________________________________________

3. x2 – 5x – 14 = 0
                      _____________________________________________________

4. 2x2 – 9x + 8 = 0
                      _____________________________________________________

5. 3x2 - 5x – 2 = 0
                      _____________________________________________________

6. x2 – 8x – 9 = 0
                      _____________________________________________________




                                                                     contents            back        next
7. 2x2 – 3x – 9 = 0
                      _____________________________________________________

8. x2 + x – 2 -
                      _____________________________________________________

9. 3x2 + 2x – 8 = 0
                      _____________________________________________________

10. 16x2 – 24x + ½ = 0
                  _____________________________________________________

11. x2 – 6x + 25 = 0
                       _____________________________________________________

12. 3x2 + x – 2 = 0
                       _____________________________________________________

13. 5x2 + 11x – 8 = 0
                   _____________________________________________________

14. x2 – 8x + 16 = 0
                   _____________________________________________________

15. 4x2 – 16x + 10 = 0
                   _____________________________________________________




                                                            contents           back   next
A. Use the discriminant to determine which of the following quadratic equations have two, one or no real
   roots. Give reasons.

       1.   x2 – 5x – 5 = 0
       2.   x2 – 3x – 2 = 3x – 11
       3.   5x2 – 9x = 2x – 7
       4.   2x2 – 7x + 8 = 0
       5.   3 – 4x – 2x2 = 0
       6.   4x2 – 9x + 5 = 3x – 7

B. By using the relations between roots and coefficients, determine if the given #s are roots of the
   corresponding given equation.

       1.   6x2 – 5x + 3 = 0              (1/6, -1)
       2.   x2 + x _ 182 = 0              (13, -4)
       3.   x2 – 5x – 14 =0               (2, -7)
       4.   2x – 9x + 8 = 0               (3, 4/3)
       5.   3x2 – 5x – 2 = 0              (2, -1/3)

C. Given one roots of the equation, find the other.

       1. x2 – 8x – 9 = 0;                r= 1
       2. 2x2 – 3x – 9 = 0;               r= 3
       3. x2 + x – 2 -     =0             r=




                                                                      contents          back       next
Chapter VI




      In this chapter, students will be taught how to find solutions to quadratic equations. This lesson
assumes students are already familiar with solving simple quadratic equations by hand, and that they have
become relatively comfortable using their graphing calculator for solving arithmetic problems and simple
algebra problems. Students will also be shown strategies on how to use the keys on the graphing calculator to
show a complete graph.


  TARGET SKILLS:
        At the end of this chapter, students are expected to:

               • use calculator in solving quadratic equation;
               • solve equation on a calculator; and.
               • improve skills using calculator by solving quadratic equation.




                                                                          contents         back       next
Module in solving quadratic equation
Module in solving quadratic equation
Module in solving quadratic equation
Module in solving quadratic equation
Module in solving quadratic equation
Module in solving quadratic equation

Más contenido relacionado

La actualidad más candente

Factoring Polynomials with common monomial factor
Factoring Polynomials with common monomial factorFactoring Polynomials with common monomial factor
Factoring Polynomials with common monomial factorLorie Jane Letada
 
Math 9 Curriculum Guide rev.2016
Math 9 Curriculum Guide rev.2016Math 9 Curriculum Guide rev.2016
Math 9 Curriculum Guide rev.2016Chuckry Maunes
 
Math Performance task (2nd quarter)
Math Performance task (2nd quarter)Math Performance task (2nd quarter)
Math Performance task (2nd quarter)milteofe Dayandayan
 
Conceptual framework of mathematics
Conceptual framework of mathematicsConceptual framework of mathematics
Conceptual framework of mathematicsAndresBrutas
 
Lesson plan on Linear inequalities in two variables
Lesson plan on Linear inequalities in two variablesLesson plan on Linear inequalities in two variables
Lesson plan on Linear inequalities in two variablesLorie Jane Letada
 
Learning plan in mathematics
Learning plan in mathematicsLearning plan in mathematics
Learning plan in mathematicsjennytuazon01630
 
Concept of angle of elevation and depression
Concept of angle of elevation and depressionConcept of angle of elevation and depression
Concept of angle of elevation and depressionJunila Tejada
 
Math10 curriculum map docx
Math10 curriculum map docxMath10 curriculum map docx
Math10 curriculum map docxEmaEmitsCP
 
Detailed Lesson Plan in Mathematics
Detailed Lesson Plan in MathematicsDetailed Lesson Plan in Mathematics
Detailed Lesson Plan in MathematicsAbbygale Jade
 
Curriculum Map in Mathematics 10 - Unit 1 (with Table of Specification and Pe...
Curriculum Map in Mathematics 10 - Unit 1 (with Table of Specification and Pe...Curriculum Map in Mathematics 10 - Unit 1 (with Table of Specification and Pe...
Curriculum Map in Mathematics 10 - Unit 1 (with Table of Specification and Pe...Lester Tiempo
 
Lesson plan about statistics math 7
Lesson plan about statistics math 7Lesson plan about statistics math 7
Lesson plan about statistics math 7Jaybie09
 
Rectangular Coordinate System Lesson Plan
Rectangular Coordinate System Lesson PlanRectangular Coordinate System Lesson Plan
Rectangular Coordinate System Lesson PlanRealyn Magbanua
 
Detailed lesson plan of Similar Triangles in Inductive Method
Detailed lesson plan of Similar Triangles in Inductive MethodDetailed lesson plan of Similar Triangles in Inductive Method
Detailed lesson plan of Similar Triangles in Inductive MethodLorie Jane Letada
 

La actualidad más candente (20)

Lp (similar polygons)
Lp (similar polygons)Lp (similar polygons)
Lp (similar polygons)
 
Factoring Polynomials with common monomial factor
Factoring Polynomials with common monomial factorFactoring Polynomials with common monomial factor
Factoring Polynomials with common monomial factor
 
Math 9 Curriculum Guide rev.2016
Math 9 Curriculum Guide rev.2016Math 9 Curriculum Guide rev.2016
Math 9 Curriculum Guide rev.2016
 
Math Performance task (2nd quarter)
Math Performance task (2nd quarter)Math Performance task (2nd quarter)
Math Performance task (2nd quarter)
 
Conceptual framework of mathematics
Conceptual framework of mathematicsConceptual framework of mathematics
Conceptual framework of mathematics
 
Lesson plan on Linear inequalities in two variables
Lesson plan on Linear inequalities in two variablesLesson plan on Linear inequalities in two variables
Lesson plan on Linear inequalities in two variables
 
Problems involving sets
Problems involving setsProblems involving sets
Problems involving sets
 
Learning plan in mathematics
Learning plan in mathematicsLearning plan in mathematics
Learning plan in mathematics
 
Concept of angle of elevation and depression
Concept of angle of elevation and depressionConcept of angle of elevation and depression
Concept of angle of elevation and depression
 
Solving Problems Using Venn Diagram
Solving Problems Using Venn DiagramSolving Problems Using Venn Diagram
Solving Problems Using Venn Diagram
 
Law of sines
Law of sinesLaw of sines
Law of sines
 
A detailed lesson plan in permutation
A detailed lesson plan in permutationA detailed lesson plan in permutation
A detailed lesson plan in permutation
 
The Law of Cosines demo
The Law of Cosines demoThe Law of Cosines demo
The Law of Cosines demo
 
Math10 curriculum map docx
Math10 curriculum map docxMath10 curriculum map docx
Math10 curriculum map docx
 
Detailed Lesson Plan in Mathematics
Detailed Lesson Plan in MathematicsDetailed Lesson Plan in Mathematics
Detailed Lesson Plan in Mathematics
 
Subsets of A Line
Subsets of A LineSubsets of A Line
Subsets of A Line
 
Curriculum Map in Mathematics 10 - Unit 1 (with Table of Specification and Pe...
Curriculum Map in Mathematics 10 - Unit 1 (with Table of Specification and Pe...Curriculum Map in Mathematics 10 - Unit 1 (with Table of Specification and Pe...
Curriculum Map in Mathematics 10 - Unit 1 (with Table of Specification and Pe...
 
Lesson plan about statistics math 7
Lesson plan about statistics math 7Lesson plan about statistics math 7
Lesson plan about statistics math 7
 
Rectangular Coordinate System Lesson Plan
Rectangular Coordinate System Lesson PlanRectangular Coordinate System Lesson Plan
Rectangular Coordinate System Lesson Plan
 
Detailed lesson plan of Similar Triangles in Inductive Method
Detailed lesson plan of Similar Triangles in Inductive MethodDetailed lesson plan of Similar Triangles in Inductive Method
Detailed lesson plan of Similar Triangles in Inductive Method
 

Destacado

Quadratic equations
Quadratic equationsQuadratic equations
Quadratic equationsA M
 
Grade 9: Mathematics Unit 1 Quadratic Equations and Inequalities.
Grade 9: Mathematics Unit 1 Quadratic Equations and Inequalities.Grade 9: Mathematics Unit 1 Quadratic Equations and Inequalities.
Grade 9: Mathematics Unit 1 Quadratic Equations and Inequalities.Paolo Dagaojes
 
Grade 9 Mathematics Module 5 Quadrilaterals (LM)
Grade 9 Mathematics Module 5 Quadrilaterals (LM)Grade 9 Mathematics Module 5 Quadrilaterals (LM)
Grade 9 Mathematics Module 5 Quadrilaterals (LM)Paolo Dagaojes
 
Quadratic Equation solved by Square root property
Quadratic Equation solved by Square root propertyQuadratic Equation solved by Square root property
Quadratic Equation solved by Square root propertyReynz Anario
 
Quadratic equation
Quadratic equation   Quadratic equation
Quadratic equation HOME!
 
Simplifying radical expressions, rational exponents, radical equations
Simplifying radical expressions, rational exponents, radical equationsSimplifying radical expressions, rational exponents, radical equations
Simplifying radical expressions, rational exponents, radical equationsJessica Garcia
 
Quadratic Equation
Quadratic EquationQuadratic Equation
Quadratic Equationitutor
 
9 2power Of Power
9 2power Of Power9 2power Of Power
9 2power Of Powertaco40
 
Quadratic function
Quadratic functionQuadratic function
Quadratic functionvickytg123
 
Digital textbook -EXPONENTS AND POWERS
Digital textbook -EXPONENTS AND POWERSDigital textbook -EXPONENTS AND POWERS
Digital textbook -EXPONENTS AND POWERSGANESHKRISHNANG
 
55 addition and subtraction of rational expressions
55 addition and subtraction of rational expressions 55 addition and subtraction of rational expressions
55 addition and subtraction of rational expressions alg1testreview
 
Module 2 similarity
Module 2   similarityModule 2   similarity
Module 2 similaritydionesioable
 
Addition and Subtraction of radicals (Dissimilar radicals)
Addition and Subtraction of radicals (Dissimilar radicals)Addition and Subtraction of radicals (Dissimilar radicals)
Addition and Subtraction of radicals (Dissimilar radicals)brixny05
 
Quadratic inequalities
Quadratic inequalitiesQuadratic inequalities
Quadratic inequalitiesmstf mstf
 
Mathematics in the Modern World
Mathematics in the Modern WorldMathematics in the Modern World
Mathematics in the Modern WorldKylyn Albay
 
Power of Power Exponent Rule
Power of Power Exponent RulePower of Power Exponent Rule
Power of Power Exponent RulePassy World
 
Lect w2 measures_of_location_and_spread
Lect w2 measures_of_location_and_spreadLect w2 measures_of_location_and_spread
Lect w2 measures_of_location_and_spreadRione Drevale
 
Similar Triangles
Similar TrianglesSimilar Triangles
Similar TrianglesPassy World
 

Destacado (20)

Quadratic equations
Quadratic equationsQuadratic equations
Quadratic equations
 
Grade 9: Mathematics Unit 1 Quadratic Equations and Inequalities.
Grade 9: Mathematics Unit 1 Quadratic Equations and Inequalities.Grade 9: Mathematics Unit 1 Quadratic Equations and Inequalities.
Grade 9: Mathematics Unit 1 Quadratic Equations and Inequalities.
 
Grade 9 Mathematics Module 5 Quadrilaterals (LM)
Grade 9 Mathematics Module 5 Quadrilaterals (LM)Grade 9 Mathematics Module 5 Quadrilaterals (LM)
Grade 9 Mathematics Module 5 Quadrilaterals (LM)
 
Quadratic Equation solved by Square root property
Quadratic Equation solved by Square root propertyQuadratic Equation solved by Square root property
Quadratic Equation solved by Square root property
 
Quadratic equation
Quadratic equation   Quadratic equation
Quadratic equation
 
Simplifying radical expressions, rational exponents, radical equations
Simplifying radical expressions, rational exponents, radical equationsSimplifying radical expressions, rational exponents, radical equations
Simplifying radical expressions, rational exponents, radical equations
 
Quadratic Equation
Quadratic EquationQuadratic Equation
Quadratic Equation
 
9 2power Of Power
9 2power Of Power9 2power Of Power
9 2power Of Power
 
Quadratic equation
Quadratic equationQuadratic equation
Quadratic equation
 
Quadratic function
Quadratic functionQuadratic function
Quadratic function
 
Digital textbook -EXPONENTS AND POWERS
Digital textbook -EXPONENTS AND POWERSDigital textbook -EXPONENTS AND POWERS
Digital textbook -EXPONENTS AND POWERS
 
55 addition and subtraction of rational expressions
55 addition and subtraction of rational expressions 55 addition and subtraction of rational expressions
55 addition and subtraction of rational expressions
 
Module 2 similarity
Module 2   similarityModule 2   similarity
Module 2 similarity
 
Addition and Subtraction of radicals (Dissimilar radicals)
Addition and Subtraction of radicals (Dissimilar radicals)Addition and Subtraction of radicals (Dissimilar radicals)
Addition and Subtraction of radicals (Dissimilar radicals)
 
Quadratic inequalities
Quadratic inequalitiesQuadratic inequalities
Quadratic inequalities
 
Math 9 (module 3)
Math 9 (module 3)Math 9 (module 3)
Math 9 (module 3)
 
Mathematics in the Modern World
Mathematics in the Modern WorldMathematics in the Modern World
Mathematics in the Modern World
 
Power of Power Exponent Rule
Power of Power Exponent RulePower of Power Exponent Rule
Power of Power Exponent Rule
 
Lect w2 measures_of_location_and_spread
Lect w2 measures_of_location_and_spreadLect w2 measures_of_location_and_spread
Lect w2 measures_of_location_and_spread
 
Similar Triangles
Similar TrianglesSimilar Triangles
Similar Triangles
 

Similar a Module in solving quadratic equation

Similar a Module in solving quadratic equation (20)

Workbook in polygons and space figures
Workbook in polygons and space figuresWorkbook in polygons and space figures
Workbook in polygons and space figures
 
Workbook in Polygons and Space figures
Workbook in Polygons and Space figuresWorkbook in Polygons and Space figures
Workbook in Polygons and Space figures
 
Quadratic equations
Quadratic equationsQuadratic equations
Quadratic equations
 
04 foreword
04 foreword04 foreword
04 foreword
 
Module in solving polynomial
Module in solving polynomialModule in solving polynomial
Module in solving polynomial
 
005 foreword
005 foreword005 foreword
005 foreword
 
Module in solving quadratic equation
Module in solving quadratic equationModule in solving quadratic equation
Module in solving quadratic equation
 
Module in solving quadratic equation
Module in solving quadratic equationModule in solving quadratic equation
Module in solving quadratic equation
 
Module in solving quadratic equation
Module  in  solving  quadratic  equation Module  in  solving  quadratic  equation
Module in solving quadratic equation
 
Flores And De Leon
Flores And De LeonFlores And De Leon
Flores And De Leon
 
Flores And De Leon
Flores And De LeonFlores And De Leon
Flores And De Leon
 
Flores And De Leon
Flores And De LeonFlores And De Leon
Flores And De Leon
 
Flores And De Leon
Flores And De LeonFlores And De Leon
Flores And De Leon
 
02 foreword 2
02 foreword 202 foreword 2
02 foreword 2
 
Module insolving polynomial
Module insolving polynomialModule insolving polynomial
Module insolving polynomial
 
Methods of teaching Mathematics
Methods of teaching MathematicsMethods of teaching Mathematics
Methods of teaching Mathematics
 
Developing Mastery In Understanding Sentences
Developing Mastery In Understanding SentencesDeveloping Mastery In Understanding Sentences
Developing Mastery In Understanding Sentences
 
Monsalve And Layba
Monsalve And LaybaMonsalve And Layba
Monsalve And Layba
 
Monsalve And Layba
Monsalve And LaybaMonsalve And Layba
Monsalve And Layba
 
My Portfolio in EdTech 2
My Portfolio in EdTech 2My Portfolio in EdTech 2
My Portfolio in EdTech 2
 

Más de aleli ariola

Más de aleli ariola (19)

Research problem
Research problemResearch problem
Research problem
 
Field Study and Pre - Service Teaching Portfolio
Field Study and Pre - Service Teaching PortfolioField Study and Pre - Service Teaching Portfolio
Field Study and Pre - Service Teaching Portfolio
 
Research proposal
Research proposalResearch proposal
Research proposal
 
Aleli powerpoint
Aleli powerpointAleli powerpoint
Aleli powerpoint
 
0015 authors page
0015 authors page0015 authors page
0015 authors page
 
0014 references
0014 references0014 references
0014 references
 
0013 chapter vi
0013 chapter vi0013 chapter vi
0013 chapter vi
 
0012 chapter v
0012 chapter v0012 chapter v
0012 chapter v
 
0011 chapter iv
0011 chapter iv0011 chapter iv
0011 chapter iv
 
0010 chapter iii
0010 chapter iii0010 chapter iii
0010 chapter iii
 
009 chapter ii
009 chapter ii009 chapter ii
009 chapter ii
 
008 chapter i
008 chapter i008 chapter i
008 chapter i
 
007 table of contents
007 table of contents007 table of contents
007 table of contents
 
006 general objectives
006 general objectives006 general objectives
006 general objectives
 
005 introduction
005 introduction005 introduction
005 introduction
 
003 forewords
003 forewords003 forewords
003 forewords
 
004 acknowledgement
004 acknowledgement004 acknowledgement
004 acknowledgement
 
002 vmgo's
002 vmgo's002 vmgo's
002 vmgo's
 
001 cover page
001 cover page001 cover page
001 cover page
 

Último

Basic Civil Engineering first year Notes- Chapter 4 Building.pptx
Basic Civil Engineering first year Notes- Chapter 4 Building.pptxBasic Civil Engineering first year Notes- Chapter 4 Building.pptx
Basic Civil Engineering first year Notes- Chapter 4 Building.pptxDenish Jangid
 
General Principles of Intellectual Property: Concepts of Intellectual Proper...
General Principles of Intellectual Property: Concepts of Intellectual  Proper...General Principles of Intellectual Property: Concepts of Intellectual  Proper...
General Principles of Intellectual Property: Concepts of Intellectual Proper...Poonam Aher Patil
 
Making communications land - Are they received and understood as intended? we...
Making communications land - Are they received and understood as intended? we...Making communications land - Are they received and understood as intended? we...
Making communications land - Are they received and understood as intended? we...Association for Project Management
 
Magic bus Group work1and 2 (Team 3).pptx
Magic bus Group work1and 2 (Team 3).pptxMagic bus Group work1and 2 (Team 3).pptx
Magic bus Group work1and 2 (Team 3).pptxdhanalakshmis0310
 
Mixin Classes in Odoo 17 How to Extend Models Using Mixin Classes
Mixin Classes in Odoo 17  How to Extend Models Using Mixin ClassesMixin Classes in Odoo 17  How to Extend Models Using Mixin Classes
Mixin Classes in Odoo 17 How to Extend Models Using Mixin ClassesCeline George
 
PROCESS RECORDING FORMAT.docx
PROCESS      RECORDING        FORMAT.docxPROCESS      RECORDING        FORMAT.docx
PROCESS RECORDING FORMAT.docxPoojaSen20
 
Holdier Curriculum Vitae (April 2024).pdf
Holdier Curriculum Vitae (April 2024).pdfHoldier Curriculum Vitae (April 2024).pdf
Holdier Curriculum Vitae (April 2024).pdfagholdier
 
UGC NET Paper 1 Mathematical Reasoning & Aptitude.pdf
UGC NET Paper 1 Mathematical Reasoning & Aptitude.pdfUGC NET Paper 1 Mathematical Reasoning & Aptitude.pdf
UGC NET Paper 1 Mathematical Reasoning & Aptitude.pdfNirmal Dwivedi
 
Grant Readiness 101 TechSoup and Remy Consulting
Grant Readiness 101 TechSoup and Remy ConsultingGrant Readiness 101 TechSoup and Remy Consulting
Grant Readiness 101 TechSoup and Remy ConsultingTechSoup
 
How to Manage Global Discount in Odoo 17 POS
How to Manage Global Discount in Odoo 17 POSHow to Manage Global Discount in Odoo 17 POS
How to Manage Global Discount in Odoo 17 POSCeline George
 
Micro-Scholarship, What it is, How can it help me.pdf
Micro-Scholarship, What it is, How can it help me.pdfMicro-Scholarship, What it is, How can it help me.pdf
Micro-Scholarship, What it is, How can it help me.pdfPoh-Sun Goh
 
Seal of Good Local Governance (SGLG) 2024Final.pptx
Seal of Good Local Governance (SGLG) 2024Final.pptxSeal of Good Local Governance (SGLG) 2024Final.pptx
Seal of Good Local Governance (SGLG) 2024Final.pptxnegromaestrong
 
Food safety_Challenges food safety laboratories_.pdf
Food safety_Challenges food safety laboratories_.pdfFood safety_Challenges food safety laboratories_.pdf
Food safety_Challenges food safety laboratories_.pdfSherif Taha
 
Accessible Digital Futures project (20/03/2024)
Accessible Digital Futures project (20/03/2024)Accessible Digital Futures project (20/03/2024)
Accessible Digital Futures project (20/03/2024)Jisc
 
Explore beautiful and ugly buildings. Mathematics helps us create beautiful d...
Explore beautiful and ugly buildings. Mathematics helps us create beautiful d...Explore beautiful and ugly buildings. Mathematics helps us create beautiful d...
Explore beautiful and ugly buildings. Mathematics helps us create beautiful d...christianmathematics
 
How to Give a Domain for a Field in Odoo 17
How to Give a Domain for a Field in Odoo 17How to Give a Domain for a Field in Odoo 17
How to Give a Domain for a Field in Odoo 17Celine George
 
1029-Danh muc Sach Giao Khoa khoi 6.pdf
1029-Danh muc Sach Giao Khoa khoi  6.pdf1029-Danh muc Sach Giao Khoa khoi  6.pdf
1029-Danh muc Sach Giao Khoa khoi 6.pdfQucHHunhnh
 
Sociology 101 Demonstration of Learning Exhibit
Sociology 101 Demonstration of Learning ExhibitSociology 101 Demonstration of Learning Exhibit
Sociology 101 Demonstration of Learning Exhibitjbellavia9
 
Kodo Millet PPT made by Ghanshyam bairwa college of Agriculture kumher bhara...
Kodo Millet  PPT made by Ghanshyam bairwa college of Agriculture kumher bhara...Kodo Millet  PPT made by Ghanshyam bairwa college of Agriculture kumher bhara...
Kodo Millet PPT made by Ghanshyam bairwa college of Agriculture kumher bhara...pradhanghanshyam7136
 
ICT Role in 21st Century Education & its Challenges.pptx
ICT Role in 21st Century Education & its Challenges.pptxICT Role in 21st Century Education & its Challenges.pptx
ICT Role in 21st Century Education & its Challenges.pptxAreebaZafar22
 

Último (20)

Basic Civil Engineering first year Notes- Chapter 4 Building.pptx
Basic Civil Engineering first year Notes- Chapter 4 Building.pptxBasic Civil Engineering first year Notes- Chapter 4 Building.pptx
Basic Civil Engineering first year Notes- Chapter 4 Building.pptx
 
General Principles of Intellectual Property: Concepts of Intellectual Proper...
General Principles of Intellectual Property: Concepts of Intellectual  Proper...General Principles of Intellectual Property: Concepts of Intellectual  Proper...
General Principles of Intellectual Property: Concepts of Intellectual Proper...
 
Making communications land - Are they received and understood as intended? we...
Making communications land - Are they received and understood as intended? we...Making communications land - Are they received and understood as intended? we...
Making communications land - Are they received and understood as intended? we...
 
Magic bus Group work1and 2 (Team 3).pptx
Magic bus Group work1and 2 (Team 3).pptxMagic bus Group work1and 2 (Team 3).pptx
Magic bus Group work1and 2 (Team 3).pptx
 
Mixin Classes in Odoo 17 How to Extend Models Using Mixin Classes
Mixin Classes in Odoo 17  How to Extend Models Using Mixin ClassesMixin Classes in Odoo 17  How to Extend Models Using Mixin Classes
Mixin Classes in Odoo 17 How to Extend Models Using Mixin Classes
 
PROCESS RECORDING FORMAT.docx
PROCESS      RECORDING        FORMAT.docxPROCESS      RECORDING        FORMAT.docx
PROCESS RECORDING FORMAT.docx
 
Holdier Curriculum Vitae (April 2024).pdf
Holdier Curriculum Vitae (April 2024).pdfHoldier Curriculum Vitae (April 2024).pdf
Holdier Curriculum Vitae (April 2024).pdf
 
UGC NET Paper 1 Mathematical Reasoning & Aptitude.pdf
UGC NET Paper 1 Mathematical Reasoning & Aptitude.pdfUGC NET Paper 1 Mathematical Reasoning & Aptitude.pdf
UGC NET Paper 1 Mathematical Reasoning & Aptitude.pdf
 
Grant Readiness 101 TechSoup and Remy Consulting
Grant Readiness 101 TechSoup and Remy ConsultingGrant Readiness 101 TechSoup and Remy Consulting
Grant Readiness 101 TechSoup and Remy Consulting
 
How to Manage Global Discount in Odoo 17 POS
How to Manage Global Discount in Odoo 17 POSHow to Manage Global Discount in Odoo 17 POS
How to Manage Global Discount in Odoo 17 POS
 
Micro-Scholarship, What it is, How can it help me.pdf
Micro-Scholarship, What it is, How can it help me.pdfMicro-Scholarship, What it is, How can it help me.pdf
Micro-Scholarship, What it is, How can it help me.pdf
 
Seal of Good Local Governance (SGLG) 2024Final.pptx
Seal of Good Local Governance (SGLG) 2024Final.pptxSeal of Good Local Governance (SGLG) 2024Final.pptx
Seal of Good Local Governance (SGLG) 2024Final.pptx
 
Food safety_Challenges food safety laboratories_.pdf
Food safety_Challenges food safety laboratories_.pdfFood safety_Challenges food safety laboratories_.pdf
Food safety_Challenges food safety laboratories_.pdf
 
Accessible Digital Futures project (20/03/2024)
Accessible Digital Futures project (20/03/2024)Accessible Digital Futures project (20/03/2024)
Accessible Digital Futures project (20/03/2024)
 
Explore beautiful and ugly buildings. Mathematics helps us create beautiful d...
Explore beautiful and ugly buildings. Mathematics helps us create beautiful d...Explore beautiful and ugly buildings. Mathematics helps us create beautiful d...
Explore beautiful and ugly buildings. Mathematics helps us create beautiful d...
 
How to Give a Domain for a Field in Odoo 17
How to Give a Domain for a Field in Odoo 17How to Give a Domain for a Field in Odoo 17
How to Give a Domain for a Field in Odoo 17
 
1029-Danh muc Sach Giao Khoa khoi 6.pdf
1029-Danh muc Sach Giao Khoa khoi  6.pdf1029-Danh muc Sach Giao Khoa khoi  6.pdf
1029-Danh muc Sach Giao Khoa khoi 6.pdf
 
Sociology 101 Demonstration of Learning Exhibit
Sociology 101 Demonstration of Learning ExhibitSociology 101 Demonstration of Learning Exhibit
Sociology 101 Demonstration of Learning Exhibit
 
Kodo Millet PPT made by Ghanshyam bairwa college of Agriculture kumher bhara...
Kodo Millet  PPT made by Ghanshyam bairwa college of Agriculture kumher bhara...Kodo Millet  PPT made by Ghanshyam bairwa college of Agriculture kumher bhara...
Kodo Millet PPT made by Ghanshyam bairwa college of Agriculture kumher bhara...
 
ICT Role in 21st Century Education & its Challenges.pptx
ICT Role in 21st Century Education & its Challenges.pptxICT Role in 21st Century Education & its Challenges.pptx
ICT Role in 21st Century Education & its Challenges.pptx
 

Module in solving quadratic equation

  • 1. contents next
  • 2. VISION A premier university in CALABARZON, offering academic programs and related services designed to respond to the requirements of the Philippines and the global economy, particularly in Asian Countries. contents back next
  • 3. MISSION The University shall primarily provide advanced education, professional, technological and vocational instruction in agriculture, fisheries, forestry, science, engineering, in dustrial technologies, teacher education, medicine, law, arts and sciences, information technologies and other related fields. It shall also undertake research and extension services and provide progressive leadership in its areas of specialization. contents back next
  • 4. GOALS In pursuit of the college vision/mission the College of Education is committed to develop the full potentials of the individuals and equip them with knowledge, skills and attitudes in Teacher Education allied fields to effectively respond to the increasing demands, challenges and opportunities of changing time for global competitiveness. contents back next
  • 5. OBJECTIVES OF BSED Produce graduate who can demonstrate and practice the professional and ethical requirement for the Bachelor of Secondary Education such as: 1.To serve as positive and powerful role models in the pursuit of the learning thereby maintaining high regards to professional growth. 2. Focus on the significance of providing wholesome and desirable learning environment. 3. Facilitate learning process in diverse type of learners. 4. Used varied learning approaches and activities, instructional materials and learning resources. 5. Used assessment data, plan and revise teaching – learning plans. 6. Direct and strengthen the links between school and community activities. 7. Conduct research and development in Teacher Education and other related activities. contents back next
  • 6. This Teacher’s “MODULE IN SOLVING QUADRATIC EQUATION” is part of the requirements in Educational Technology 2 under the revised curriculum based on CHED Memorandum Order (CMO)-30, Series of 2004. Educational Technology 2 is a three (3)-unit course designed to introduce both traditional and innovative technologies to facilitate and foster meaningful and effective learning where students are expected to demonstrate a sound understanding of the nature, application and production of the various types of educational technologies. Students are provided with guidance and assistance of selected faculty The members of the College on the selection, production and utilization of appropriate technology tools in developing technology-based teacher support materials. Through the role and functions of computers especially the Internet, the student researchers and the advisers are able to design and develop various types of alternative delivery systems. These kinds of activities offer a remarkable learning experience for the education students as future mentors especially in the preparation and utilization of instructional materials. The output of the group’s effort may serve as a contribution to the existing body instructional materials that the institution may utilize in order to provide effective and quality education. The lessons and evaluations presented in this module may also function as a supplementary reference for secondary teachers and students. reference for secondary teachers and students. Aleli M. Ariola Module Developer Shane Maureen D. Atendido Module Developer contents back next
  • 7. This Teacher’s “MODULE IN SOLVING QUADRATIC EQUATION” is part of the requirements in Educational Technology 2 under the revised curriculum based on CHED Memorandum Order (CMO)-30, Series of 2004. Educational Technology 2 is a three (3)-unit course designed to introduce both traditional and innovative technologies to facilitate and foster meaningful and effective learning where students are expected to demonstrate a sound understanding of the nature, application and production of the various types of educational technologies. Students are provided with guidance and assistance of selected faculty The members of the College through the selection, production and utilization of appropriate technology tools in developing technology-based teacher support materials. Through the role and functions of computers especially the Internet, the student researchers and the advisers are able to design and develop various types of alternative delivery systems. These kinds of activities offer a remarkable learning experience for the education students as future mentors especially in the preparation and utilization of instructional materials. The output of the group’s effort may serve as a contribution to the existing body instructional materials that the institution may utilize in order to provide effective and quality education. The lessons and evaluations presented in this module may also function as a supplementary reference for secondary teachers and students. reference for secondary teachers and students. FOR-IAN V. SANDOVAL Computer Instructor / Adviser Educational Technology 2 DELIA F. MERCADO Module Consultant / Instructor 3 Principal of Laboratory High School LYDIA R. CHAVEZ Dean College of Education contents back next
  • 8. The authors would like to acknowledge with deep appreciation and gratitude the invaluable help of the following persons: Mr. For-Ian V. Sandoval our module adviser, Computer Instructor / Adviser Educational Technology 2 for giving us opportunity to participate on this project, and for guiding us and pursue us to finish this module. Mrs. Delia F. Mercado, Instructor III and Director of Laboratory High School, for being our Teacher Consultant for the completion of this modular workbook. Mrs. Corazon San Agustin, our instructor in Educational Technology I, for giving us guidance and encouragement us in completing the requirement. Mrs. Lydia Chavez, Dean of Education for the support and guidance. We also wish to thank our family and friends as an inspiration and understand us they were robbed of many precious moments as we looked ourselves in our rooms when our minds went prolific and our hands itched to write. And finally, we thank Almighty God, the source of all knowledge, understanding and wisdom. From him we owe all that we have and all that we are! Once again, we thank all those who have encourage and helped us in preparing this module for publication and who have extended us much understanding, patience, and support. THE AUTHORS contents back next
  • 9. A quadratic equation is a second-order polynomial equation in a single variable x. The general form is where x represents a variable, and a, b, and c, represent coefficients and constants, with a ≠ 0. (If a = 0, the equation becomes a linear equation.) Among his many other talents, Major General Stanley in Gilbert and Sullivan's operetta the Pirates of Penzance impresses the pirates with his knowledge of quadratic equations in "The Major General's Song" as follows: "I am the very model of a modern Major-General, I've information vegetable, animal, and mineral, I know the kings of England, and I quote the fights historical, From Marathon to Waterloo, in order categorical; I'm very well acquainted too with matters mathematical, I understand equations, both the simple and quadratic, About binomial theorem I'm teeming with a lot o' news-- With many cheerful facts about the square of the hypotenuse." The constants a, b, and c, are called respectively, the quadratic coefficient, the linear coefficient and the constant term or free term. Quadratic comes from quadratus, which is the Latin word for "square." Quadratic equations can be solved by factoring, completing the square, graphing, Newton's method, and using the quadratic formula. One common use of quadratic equations is computing trajectories in projectile motion. This module centers on the different ways of solving the quadratic equation by factoring, by finding square roots, by completing the square, and by using the quadratic formula. Students are given guides to determine the most appropriate method to use. Identifying the disciminant of the quadratic equation and finding the relationship between the coefficient and the root of the quadratic equation are also discussed. contents back next
  • 10. At the end of this module, students are expected to: •distinguish what is quadratic equation and complex numbers; •recognize property of the number I and solutions in solving the four arithmetic operations; •solve quadratic equation by factoring, completing the square, quadratic formula and solving by graphing; •learn the technique on how to use any method for solving quadratic equation, taking the square root and transforming quadratic equations by appropriate substitution; •determine discriminant, and relations between roots and coefficient; and •use calculator in solving quadratic equation. contents back next
  • 11. VMGO’s of BSEd Foreword Acknowledgement Introduction General Objective’s Table of Contents Chapter I. Identify the Quadratic Equation Lesson 1. Quadratic Equation Chapter II. Complex Number Lesson 2. Defining Complex Number Lesson 3. Number i Lesson 4. Complex Plane Lesson 5. Complex Arithmetic Chapter III. Solving Quadratic Equation Lesson 6. Factoring Lesson 7. Completing the Square Lesson 8. Quadratic Formula Lesson 9. Solving by Graphing contents back next
  • 12. Chapter IV. Solving Equation on Quadratic Lesson 10. Equation in Quadratic Form Lesson 11. Equation Containing Radicals Lesson 12. Equation Reducible to Quadratic Equation Chapter V. The Discriminant, Roots and Coefficient Lesson 13. Discriminant and the Roots of a Quadratic Equation Lesson 14. Relation between Roots and Coefficient Chapter VI. Solving Quadratic Equation on a Calculator Lesson 15. Equation on a Calculator References Demo Slide share contents back next
  • 13. Chapter I This chapter deals with equations which are classified according to the highest power of its variable. An equation in the variable x whose highest power is 2 is called a quadratic equation. It will be observed here that variable a, b and c are real numbers and a cannot be 0. TARGET SKILLS: At the end of this chapter, students are expected to: • identify quadratic equation; • discuss real numbers and standard form of the quadratic equation; • express quadratic equation in standard form; and • apply distributive property in solving quadratic equation. contents back next
  • 14. Lesson 1 Identifying the quadratic equation OBJECTIVES: At the end of this lesson, students are expected to: •define the quadratic equation; •discuss real numbers in quadratic equation; and •improve writing the standard form of the quadratic equation. Polynomials are classified according to the highest power of its variable. A first degree polynomial, like 2x + 5 is linear; a second degree polynomial, like x2 + 2 – 3 is quadratic; a third degree polynomial, like x3 + 4x2 – 3x + 12 is cubic. Similarly, equation and inequalities are classified according to the highest power of its variable. An equation in the variable x whose highest power is two is called a quadratic equation. Some examples are x2 – 64, 4n2 = 25, 3x2 – 4x + 1 = 0. An equation of the form ax2 + bx + c = 0, where a, b and c are constant and a not equal to 0, a id a quadratic equation. to 0, a id a quadratic equation. Any quadratic equation can be written in the form ax2 + bx + c = 0. This is also called the standard form of the quadratic equation. Here, a, b and c are real numbers and a cannot be 0. Example A. Express x2 = 8x in standard form x2 = 8x can be written as x2 - 8x = 0 where a=1, b= ˉ8, and c=0. Example B. Express x2 = 64 in standard form contents back next
  • 15. x2 = 64 can be written as x2 – 64 = 0 where a=1. B=0, and c=ˉ64. Example C. Express the fractional equation x = 1/x-3 as a quadratic equation. x = 1/x-3 x (x-3) = 1 multiply both sides by x-3 x² - 3x = 1 using the distributive property x² - 3x - 1 = 0 a=1, b=ˉ3, c=ˉ1 Exercises: Which of the following equations are quadratic? 1.3x = x² - 5 2. 2x =1 3. x² = 25 4. 2x - 3 = x + 5 5. 5x – 2y = 0 contents back next
  • 16. Name: ___________________ Section: _______ Instructor: ________________ Date: _______ Rating: _______ Instruction: Write the following equations in the form ax2 + bx + c = 0, and give the value of a, b, and c. 1. x2 = 6x _____________________________________________ 2. 2x2 = 32 _____________________________________________ 3. 3x2 = 5x – 1 _____________________________________________ 4. 10 = 3x – x2 _____________________________________________ 5. (x + 2)2 = 9 _____________________________________________ 6. 4x2 = 64 _____________________________________________ contents back next
  • 17. 7. _____________________________________________ 8. _____________________________________________ 9. 8x = x2 _____________________________________________ 2 10. =6 _____________________________________________ 11. _____________________________________________ 12. x2 = _____________________________________________ 13. _____________________________________________ 14. x2 + _____________________________________________ 15. (x + 1)(x-3) = 6 _____________________________________________ contents back next
  • 18. A. Define each of the following terms. 1. Quadratic equation 2. Standard form of a quadratic equation 3. Real numbers B. Which of the following equations are quadratic? 1. 4x = 2x2 – 6 2. 3x = 1 3. 5x2 = 30 4. 3x – 2 = 2x + 6 5. 2x – 5y = 0 6. 4x + 2x2 – 3x3 = 0 7. 12x2 – x = 11 C. Write the following equations in the form ax2 + bx + c = 0, and give the values of a, b and c. 1. 3x2 = 6x 2. 3x2 = 32 3. 2x2 = 5x – 1 4. 12 = 4x – x2 5. (x + 3)2 = 8 6. 4x2 = 56 7. 1/x + x = 6 8. x(x – 4) – 1 = 0 9. 9x = x2 10. (1/x)2 = 10 contents back next
  • 19. Chapter II This chapter centers on the complex numbers which is a number comprising a real number and an imaginary number. Under this, we have the number i, the complex plane where the points are plotted and the 4 arithmetic operations such as addition and subtraction, multiplication and division of complex numbers. To round up the chapter, simple equation involving complex numbers will be studied and solved. TARGET SKILLS: At the end of this chapter, students are expected to: • identify complex numbers; • differentiate the real pat and imaginary part of complex numbers; and • explore solving of the 4 arithmetic operations on the complex numbers. contents back next
  • 20. Lesson 2 Defining Complex Numbers OBJECTIVES: At the end of this lesson, students are expected to: •identify complex numbers; •differentiate the real number and standard imaginary unit; and •extend the ordinary real number. A complex number, in mathematics, is a number comprising a real number and an imaginary number; it can be written in the form a + bi, where a and b are real numbers, and i is the standard imaginary unit, having the property that i2 = −1. The complex numbers contain the ordinary real numbers, but extend them by adding in extra numbers and correspondingly expanding the understanding of addition and multiplication. Equation 1: x2 - 1 = 0. Equation 1 has two solutions, x = -1 and x = 1. We know that solving an equation in x is equivalent to finding the x-intercepts of a graph; and, the graph of y = x2 - 1 crosses the x-axis at (-1,0) and (1,0). contents back next
  • 21. Equation 2: x2 + 1 = 0 Equation 2 has no solutions, and we can see this by looking at the graph of y = x2 + 1. Since the graph has no x-intercepts, the equation has no solutions. When we define complex numbers, equation 2 will have two solutions. contents back next
  • 22. Name: ___________________ Section: _______ Instructor: ________________ Date: ________ Rating: _______ Solve each equation and graph. 1. x² + 4 = 0 _____________________________________________ 2. 2x² + 18 = 0 _____________________________________________ 3. 2x² + 14 = 0 _____________________________________________ 4. 3x² + 27 = 0 _____________________________________________ 5. x² - 3 = 0 _____________________________________________ 6. x² + 21 = 0 _____________________________________________ contents back next
  • 23. 7. 3x² - 5 = 0 _____________________________________________ 8. 5x² + 30 = 0 _____________________________________________ 9. 2x² + 3 = 0 _____________________________________________ 10. x² + 50 = 0 _____________________________________________ 11. x² - 2 = 0 _____________________________________________ 12. 3x² - 50 = 0 _____________________________________________ 13. x² - 3 = 0 _____________________________________________ 14. x² + 4 = 0 _____________________________________________ 15. 2x² + 14 = 0 _____________________________________________ contents back next
  • 24. Lesson 3 The Number i OBJECTIVES: At the end of this lesson, students are expected to: •recognize the property of the number i; •discuss the powers of i; and •solve the high powers of imaginary unit. Consider Equations 1 and 2 again. Equation 1 Equation 2 x2 - 1 = 0. x2 + 1 = 0. x2 = 1. x2 = -1. Equation 1 has solutions because the number 1 has two square roots, 1 and -1. Equation 2 has no solutions because -1 does not have a square root. In other words, there is no number such that if we multiply it by itself we get - 1. If Equation 2 is to be given solutions, then we must create a square root of -1. The imaginary unit i is defined by The definition of i tells us that i2 = -1. We can use this fact to find other powers of i. contents back next
  • 25. Example i3 = i2 * i = -1*i = -i. i4 = i2 * i2 = (-1) * (-1) = 1. Exercise: Simplify i8 and i11. We treat i like other numbers in that we can multiply it by numbers, we can add it to other numbers, etc. The difference is that many of these quantities cannot be simplified to a pure real number. For example, 3i just means 3 times i, but we cannot rewrite this product in a simpler form, because it is not a real number. The quantity 5 + 3i also cannot be simplified to a real number. However, (-i)2 can be simplified. (-i)2 = (-1*i)2 = (-1)2 * i2 = 1 * (-1) = -1. Because i2 and (-i)2 are both equal to -1, they are both solutions for Equation 2 above. contents back next
  • 26. Name: ___________________ Section: _______ Instructor: ________________ Date: _______ Rating: ______ Instruction: Express each number in terms of i and simplify. 1. ______________________________________________________ 2. ______________________________________________________ 3. ______________________________________________________ 4. ______________________________________________________ 5. ______________________________________________________ contents back next
  • 27. 6. ______________________________________________________ 7. ______________________________________________________ 8. ______________________________________________________ 9. ______________________________________________________ 10. ______________________________________________________ 11. ______________________________________________________ 12. ______________________________________________________ 13. ______________________________________________________ contents back next
  • 28. Lesson 4 The Complex Plane OBJECTIVES: At the end of this lesson, students are expected to: •distinguish the points on the plane; •differentiate the real and imaginary part; and •draw from memory the figure form by the plot points on the complex plane. A complex number is one of the form a + bi, where a and b are real numbers. a is called the real part of the complex number, and b is called the imaginary part. Two complex numbers are equal if and only if their real parts are equal and their imaginary parts are equal. I.e., a+bi = c+di if and only if a = c, and b = d. Example. 2 - 5i. 6 + 4i. 0 + 2i = 2i. 4 + 0i = 4. The last example above illustrates the fact that every real number is a complex number (with imaginary part 0). Another example: the real number -3.87 is equal to the complex number -3.87 + 0i. It is often useful to think of real numbers as points on a number line. For example, you can define the order relation c < d, where c and d are real numbers, by saying that it means c is to the left of d on the number line. contents back next
  • 29. We can visualize complex numbers by associating them with points in the plane. We do this by letting the number a + bi correspond to the point (a,b), we use x for a and y for b. Exercises: Represent each of the following complex number by a point in the plane. 1. 3 + 2i 2. 1 – 4i 3. 4 + 3i 4. 2 – 5i 5. 4 – 3i contents back next
  • 30. Name: ___________________ Section: _______ Instructor: ________________ Date: _______ Rating: ______ Instruction: Represent each of the following Complex Numbers by a point in the plane. 1. ______________________________________________________ 2. ______________________________________________________ 3. ______________________________________________________ 4. 0 ______________________________________________________ 5. 3 ______________________________________________________ 6. ______________________________________________________ 7. 1/2 ______________________________________________________ 8. ______________________________________________________ contents back next
  • 31. 9. _____________________________________________________ 10. ____________________________________________________ 11. _____________________________________________________ 12. _____________________________________________________ 13. _____________________________________________________ 14. _____________________________________________________ 15. _____________________________________________________ contents back next
  • 32. Lesson 5 Complex Arithmetic OBJECTIVES: At the end of this lesson, students are expected to: •define the four arithmetic operations on complex numbers; •comply with the steps in solving the different operations; and •solve the four arithmetic operations. When a number system is extended the arithmetic operations must be defined for the new numbers, and the important properties of the operations should still hold. For example, addition of whole numbers is commutative. This means that we can change the order in which two whole numbers are added and the sum is the same: 3 + 5 = 8 and 5 + 3 = 8. We need to define the four arithmetic operations on complex numbers. Addition and Subtraction To add or subtract two complex numbers, you add or subtract the real parts and the imaginary parts. (a + bi) + (c + di) = (a + c) + (b + d)i. (a + bi) - (c + di) = (a - c) + (b - d)i. Example (3 - 5i) + (6 + 7i) = (3 + 6) + (-5 + 7)i = 9 + 2i. (3 - 5i) - (6 + 7i) = (3 - 6) + (-5 - 7)i = -3 - 12i. contents back next
  • 33. Note These operations are the same as combining similar terms in expressions that have a variable. For example, if we were to simplify the expression (3 - 5x) + (6 + 7x) by combining similar terms, then the constants 3 and 6 would be combined, and the terms -5x and 7x would be combined to yield 9 + 2x. The Complex Arithmetic applet below demonstrates complex addition in the plane. You can also select the other arithmetic operations from the pull down list. The applet displays two complex numbers U and V, and shows their sum. You can drag either U or V to see the result of adding other complex numbers. As with other graphs in these pages, dragging a point other than U or V changes the viewing rectangle. Multiplication The formula for multiplying two complex numbers is (a + bi) * (c + di) = (ac - bd) + (ad + bc)i. You do not have to memorize this formula, because you can arrive at the same result by treating the complex numbers like expressions with a variable, multiply them as usual, then simplify. The only difference is that powers of i do simplify, while powers of x do not. Example (2 + 3i)(4 + 7i) = 2*4 + 2*7i + 4*3i + 3*7*i2 = 8 + 14i + 12i + 21*(-1) = (8 - 21) + (14 + 12)i = -13 + 26i. contents back next
  • 34. Notice that in the second line of the example, the i2 has been replaced by -1. Using the formula for multiplication, we would have gone directly to the third line. Exercise Perform the following operations. (a) (-3 + 4i) + (2 - 5i) (b) 3i - (2 - 4i) (c) (2 - 7i)(3 + 4i) (d) (1 + i)(2 - 3i) Division The conjugate (or complex conjugate) of the complex number a + bi is a - bi. Conjugates are important because of the fact that a complex number times its conjugate is real; i.e., its imaginary part is zero. (a + bi)(a - bi) = (a2 + b2) + 0i = a2 + b2. Example Number Conjugate Product 2 + 3i 2 - 3i 4 + 9 = 13 3 - 5i 3 + 5i 9 + 25 = 34 4i -4i 16 contents back next
  • 35. Suppose we want to do the division problem (3 + 2i) (2 + 5i). First, we want to rewrite this as a fractional expression . Even though we have not defined division, it must satisfy the properties of ordinary division. So, a number divided by itself will be 1, where 1 is the multiplicative identity; i.e., 1 times any number is that number. So, when we multiply by, , we are multiplying by 1 and the number is not changed. Notice that the quotient on the right consists of the conjugate of the denominator over itself. This choice was made so that when we multiply the two denominators, the result is a real number. Here is the complete division problem, with the result written in standard form. contents back next
  • 36. Exercise: Write (2 - i) (3 + 2i) in standard form. We began this section by claiming that we were defining complex numbers so that some equations would have solutions. So far we have shown only one equation that has no real solutions but two complex solutions. In the next section we will see that complex numbers provide solutions for many equations. In fact, all polynomial equations have solutions in the set of complex numbers. This is an important fact that is used in many mathematical applications. Unfortunately, most of these applications are beyond the scope of this course. See your text (p. 195) for a discussion of the use of complex numbers in fractal geometry. contents back next
  • 37. Name: ___________________ Section: _______ Instructor: ________________ Date: _______ Rating: ______ Instruction: Perform the indicated operations and express the result in the form . 1. _____________________________________________________ 2. _____________________________________________________ 3. _____________________________________________________ 4. _____________________________________________________ 5. _____________________________________________________ 6. _____________________________________________________ 7. _____________________________________________________ contents back next
  • 38. 8. _____________________________________________________ 9. _____________________________________________________ 10. _____________________________________________________ 11. _____________________________________________________ 12. _____________________________________________________ 13. _____________________________________________________ 14. _____________________________________________________ 15. _____________________________________________________ contents back next
  • 39. A. Define and/or describe each of the following terms. 1. Imaginary part 2. Real number 3. Complex number 4. Complex plane 5. Imaginary unit 6. Commutative property 7. Complex conjugate B. 1. Simplify: 1. i15 2. i25 3. i106 4. i207 5. i21 2. Perform the indicated operation and express each answer. a. + b. + c. + d. + e. + f. + contents back next
  • 40. 3. Represent each complex numbers by a point in the plane. a. 3–i b. -2 + 4i c. -3 + 3i d. 4 + 5i e. -3 + 5i 4. Give the real part and the imaginary part of each complex numbers in #3. 5. Perform the indicated operations. a. (3 – 2i) + (-7 + 3i) b. (-4 + 7i) + (9 – 2i) c. (14 – 9i) + (7 – 6i) d. (5 + i) – (3 + 2i) e. (7 – 2i) – (4 – 6i) f. (8 + 3i) – (-4 – 2i) g. (3 – 2i) (3 +2i) h. (5 + 3i) (4 – i) i. (11 + 2i)2 (5 – 2i) j. (5 + 4i) / (3 – 2i) k. (4 + i) (3 – 5i) / (2 – 3i) l. (7 + 3i) / (3 – 3i / 4) contents back next
  • 41. Chapter III In this chapter, the different ways of solving the quadratic equation are recalled. There are by using the factoring, completing the square, by quadratic formula and solving by graphing. Students are given guides to determine the most appropriate method to use. TARGET SKILLS: At the end of this chapter, students are expected to: • distinguish appropriate method in solving quadratic equation; • discuss and follow the steps in such different method; and • resolve quadratic equation using any method you want. contents back next
  • 42. Lesson 6 Solving by factoring OBJECTIVES: At the end of this lesson, students are expected to: •define what is factoring; •discuss the Zero Factor Principle; and •solve equation by using the factoring method. Factoring – rearrange the equation; factor the left member; equate each factor to zero to obtain the two roots. • Solve (x – 3)(x – 4) = 0. The Zero Factor Principle tells me that at least one of the factors must be equal to zero. Since at least one of the factors must be zero, I'll set them each equal to zero: x – 3 = 0 or x – 4 = 0 x = 3 or x = 4 Solve: x = 3, 4 Note that "x = 3, 4" means the same thing as "x = 3 or x = 4"; the only difference is the formatting. The "x = 3, 4" format is more-typically used. Checking x = 3 in (x – 3)(x – 4) = 0: ([3] – 3)([3] – 4) ?=? 0 (3 – 3)(3 – 4) ?=? 0 (0)(–1) ?=? 0 0 = 0 contents back next
  • 43. Checking x = 4 in (x – 3)(x – 4) = 0: ([4] – 3)([4] – 4) ?=? 0 (4 – 3)(4 – 4) ?=? 0 (1)(0) ?=? 0 0 = 0 • Solve x2 + 5x + 6 = 0. This equation is already in the form "(quadratic) equals (zero)" but, unlike the previous example, this isn't yet factored. The quadratic must first be factored, because it is only when you MULTIPLY and get zero that you can say anything about the factors and solutions. You can't conclude anything about the individual terms of the unfactored quadratic (like the 5x or the 6), because you can add lots of stuff that totals zero. So the first thing I have to do is factor: x2 + 5x + 6 = (x + 2)(x + 3) Set this equal to zero: (x + 2)(x + 3) = 0 Solve each factor: x + 2 = 0 or x + 3 = 0 x = –2 or x = – 3 The solution to x2 + 5x + 6 = 0 is x = –3, –2 Checking x = –3 and x = –2 in x2 + 5x + 6 = 0: [–3]2 + 5[–3] + 6 ?=? 0 9 – 15 + 6 ?=? 0 9 + 6 – 15 ?=? 0 contents back next
  • 44. 15 – 15 ?=? 0 0 = 0 [–2] 2 + 5[–2] + 6 ?=? 0 4 – 10 + 6 ?=? 0 4 + 6 – 10 ?=? 0 10 – 10 ?=? 0 0 = 0 So both solutions "check". • Solve x2 – 3 = 2x. This equation is not in "(quadratic) equals (zero)" form, so I can't try to solve it yet. The first thing I need to do is get all the terms over on one side, with zero on the other side. Only then can I factor and solve: x2 – 3 = 2x x2 – 2x – 3 = 0 (x – 3)(x + 1) = 0 x – 3 = 0 or x + 1 = 0 x = 3 or x = –1 Then the solution to x2 – 3 = 2x is x = –1, 3 • Solve (x + 2)(x + 3) = 12. The (10 + 2)(9 + 3) does not equal 12, you should never forget that you must have "(quadratic) equals (zero)" before you can solve. So, tempting though it may be, the factors above equal to the other side of the equation and "solve". Instead, multiply out and simplify the left-hand side, then subtract the 12 over to the left-hand side, and re-factor. contents back next
  • 45. (x + 2)(x + 3) = 12 x2 + 5x + 6 = 12 x2 + 5x – 6 = 0 (x + 6)(x – 1) = 0 x + 6 = 0 or x – 1 = 0 x = –6 or x = 1 Then the solution to (x + 2)(x + 3) = 12 is x = –6, 1 • Solve x(x + 5) = 0. To "solve" the equation for "x + 5 = 0", divide it by x. But it can't divide by zero; dividing off the x makes the implicit assumption that x is not zero. Used the variable factors having variables and numbers (like the other factor, x + 5), a factor can contain only a variable, so "x" is a perfectly valid factor. So set the factors equal to zero, and solve: x(x + 5) = 0 x = 0 or x + 5 = 0 x = 0 or x = –5 Then the solution to x(x + 5) = 0 is x = 0, –5 • Solve x2 – 5x = 0. Factor the x out of both terms, taking the x out front. x(x – 5) = 0 x = 0 or x – 5 = 0 x = 0 or x = 5 Then the solution to x2 – 5x = 0 is x = 0, 5 There is one other case of two-term quadratics that you can factor: contents back next
  • 46. •Solve x2 – 4 = 0. This equation is in "(quadratic) equals (zero)" form, it's ready to solve. The quadratic itself is a difference of squares, then apply the difference-of-squares formula: x2 – 4 = 0 (x – 2)(x + 2) = 0 x – 2 = 0 or x + 2 = 0 x = 2 or x = –2 Then the solution is x = –2, 2 Note: This solution may also be formatted as "x = 2“ Exercises: Solve: 1.(x – 3)(x – 5) = 0. 2.x2 + 6x + 7 = 0. 3.x2 – 4 = 2x. 4.x2 – 6x = 0. 5.x2 – 8 = 0. contents back next
  • 47. Name: ___________________ Section: _______ Instructor: ________________ Date: ________ Rating: _______ Instruction: Solve the following Quadratic Equation by Factoring Method. 1. x2 – 36 = 0 _____________________________________________________ 2. x2= 25 _____________________________________________________ 3. x2 – 12x + 35 = 0 _____________________________________________________ 4. x2 – 3x – 40 = 0 _____________________________________________________ 5. 2x2 – 5x = 3 _____________________________________________________ 6. 3x2 + 25x = 18 _____________________________________________________ 7. 15x2 – 2x – 8 = 0 _____________________________________________________ contents back next
  • 48. 8. 3x2 – x = 10 _____________________________________________________ 9. x2 + 6x – 27 = 0 _____________________________________________________ 10. y2 – 2y – 3 = y – 3 _____________________________________________________ 11. 4y2 + 4y = 3 _____________________________________________________ 12. 3a2 + 10a = -3 _____________________________________________________ 13. a2 – 2a – 15 = 0 _____________________________________________________ 14. r2 + 6r – 27 = 0 _____________________________________________________ 15. 2z2 – 2 – 1 = 0 _____________________________________________________ contents back next
  • 49. Lesson 7 Solving by Completing the Square OBJECTIVES: At the end of this lesson, students are expected to: •analyze the techniques in completing the square; •comply with the techniques of completing the square; and •carefully change the exact signs for every equation. Some quadratics is fairly simple to solve because they are of the form "something-with-x squared equals some number", and then you take the square root of both sides. An example would be: (x – 4)2 = 5 x – 4 = sqrt(5) x = 4 sqrt(5) x = 4 – sqrt(5) and x = 4 + sqrt(5) Unfortunately, most quadratics doesn’t come neatly squared like this. For your average everyday quadratic, you first have to use the technique of "completing the square" to rearrange the quadratic into the neat "(squared part) equals (a number)" format demonstrated above. For example: • Find the x-intercepts of y = 4x2 – 2x – 5. First off, remember that finding the x-intercepts means setting y equal to zero and solving for the x-values, so this question is really asking you to "Solve 4x2 – 2x – 5 = 0". contents back next
  • 50. The answer can also be written in rounded form as contents back next
  • 51. You will need rounded form for "real life" answers to word problems, and for graphing. But (warning!) in most other cases, you should assume that the answer should be in "exact" form, complete with all the square roots. When you complete the square, make sure that you are careful with the sign on the x-term when you multiply by one-half. If you lose that sign, you can get the wrong answer in the end, because you'll forget what goes inside the parentheses. Also, don't be sloppy and wait to do the plus/minus sign until the very end. On your tests, you won't have the answers in the back, and you will likely forget to put the plus/minus into the answer. Besides, there's no reason to go ticking off your instructor by doing something wrong when it's so simple to do it right. On the same note, make sure you draw in the square root sign, as necessary, when you square root both sides. Don't wait until the answer in the back of the book "reminds" you that you "meant" to put the square root symbol in there. If you get in the habit of being sloppy, you'll only hurt yourself! • Solve x2 + 6x – 7 = 0 by completing the square. Do the same procedure as above, in exactly the same order. (Study tip: Always working these problems in exactly the same way will help you remember the steps when you're taking your tests.) contents back next
  • 52. If you are not consistent with remembering to put your plus/minus in as soon as you square-root both sides, then this is an example of the type of exercise where you'll get yourself in trouble. You'll write your answer as "x = –3 + 4 = 1", and have no idea how they got "x = –7", because you won't have a square root symbol "reminding" you that you "meant" to put the plus/minus in. That is, if you're sloppy, these easier problems will embarrass you! Exercise: 1. 3x2 – 4x – 6 = 0 2. 2x2 -3x + 4 = 0 3. x2 – 8x + 16 = 0 4. x2 + 18x + 72 = 0 5. 2x2 – 6x + 1 = 0 contents back next
  • 53. Name: ___________________ Section: _______ Instructor: ________________ Date: _______ Rating: ____ Instruction: Solve the following Quadratic Equation by Completing the Square. 1. x2 + 3x = 4 _____________________________________________________ 2. x2 – 2x = 24 _____________________________________________________ 3. x2 + 4 = 4x _____________________________________________________ 4. 2x2 – 6 = x _____________________________________________________ 5. 4a2 + 12a + 9 = 0 _____________________________________________________ 6. 3a2 – 5 = 14a _____________________________________________________ 7. 16b2 + 1 = 16b _____________________________________________________ contents back next
  • 54. 8. 9b2 – 6b – 1 = 0 _____________________________________________________ 9. 9z2 + 30z + 20 = 0 _____________________________________________________ 10. 2a2 + a = 10a _____________________________________________________ 11. 2x2 + 17 = 10x _____________________________________________________ 12. 2a2 + 6a + 9 = 0 _____________________________________________________ 13. 5x2 – 2x + 1 = 0 _____________________________________________________ 14. 3x2 + 2x + 1= 0 _____________________________________________________ 15. 2y2 + 5y = 42 _____________________________________________________ contents back next
  • 55. Lesson 8 Quadratic Formula OBJECTIVES: At the end of this lesson, students are expected to: •follow the step in solving quadratic formula; •distinguish the roots of the quadratic equation; and •perform substituting the values in the quadratic formula. The following steps will serve as guide in solving this method. Step 1. First subtract c from both sides of the equation and then, divide both sides by (a # 0 by hypothesis) to obtain the equivalent equation, x2 + = Step 2. Complete the left-hand side in to the perfect square. x2 + bx/a + (b/2a)2 = (b/2a)2 – c/a or (x+b/2a)2 = (b2-4ac)/4a2 Step 3. Take the square roots of both sides of the last equation. (x+b/2a) = ( /2a contents back next
  • 56. Step 4. Solve for x. x == or Let a, b and c be real constant, where a ≠ 0. Then the roots of ax2 + bx + c = 0 are = x= The above formula is referred to as the quadratic formula. Example: Solve a. 3x2 – x – 5/2 = 0 Solutions: Here a=3, b=⁻1, c=⁻5/2 Substituting these values in the quadratic formula we obtain x = = = The roots are and . a. 2x2 – 5 (x-2) = 8 To be able to apply the formula, we must first put the given equation in standard form. 2x2 – 5 (x-2) = 8 2x2 – 5x + 10 = 8 2x2 – 5x + 2 = 0 contents back next
  • 57. Here a=2, b=⁻5 c=2. By the quadratic formula . x= = The roots are 2 and . Note that the expression 2x2 – 5x + 2 can be factored as 2x2 – 5x + 2 = (2x – 1) (x – 2) The roots of the quadratic equation x = 1/2 and x = 2. This example shown that if we can see that the given equation in factorable, it will be quicker to solve it by factoring. Exercises: Solve each equation by quadratic formula. 1. x2 – 14x + 49 = 0 2. x2 – 4x – 21 = 0 3. x2 + 5x – 36 = 0 4. x2 + x – 30 = 0 5. x2 + 3x = 40 contents back next
  • 58. Name: ___________________ Section: _______ Instructor: ________________ Date: ________ Rating: ______ Instruction: Solve the following equations by the Quadratic Formula. 1. 2a2 – 10 = 9 _____________________________________________________ 2. 6b2 – b = 12 _____________________________________________________ 3. 3x2 + x = 14 _____________________________________________________ 4. 10a2 + 3 = 11a _____________________________________________________ 5. 2x2 + 5x = 12 _____________________________________________________ 6. 4x2 + 5x = 21 _____________________________________________________ contents back next
  • 59. 7. 2x2 – 7x + 3 = 0 _____________________________________________________ 8. 3a2 – 6a + 2 = 0 _____________________________________________________ 9. 3b2 – 2b – 4 = 0 _____________________________________________________ 10 a2 – 3a – 40 = 0 _____________________________________________________ 11. 3y2 – 11y + 10 = 0 _____________________________________________________ 12. 3w2 = 9 + 2w _____________________________________________________ 13. 15z2 + 22z = 48 _____________________________________________________ 14. 9a2 + 14 = 24a _____________________________________________________ 15. 16m2 = 24m + 19 _____________________________________________________ contents back next
  • 60. Lesson 9 Solving "by Graphing OBJECTIVES: At the end of this lesson, students are expected to: •define graphing; •resolve the equation by graphing; and •draw the points from the equations given. To be honest, solving "by graphing" is an achingly trendy but somewhat bogus topic. The basic idea behind solving by graphing is that, since the "solutions" to "ax2 + bx + c = 0" are the x-intercepts of "y = ax2 + bx + c", you can look at the x-intercepts of the graph to find the solutions to the equation. There are difficulties with "solving" this way, though.... When you graph a straight line like "y = 2x + 3", you can find the x-intercept (to a certain degree of accuracy) by drawing a really neat axis system, plotting a couple points, grabbing your ruler and drawing a nice straight line, and reading the (approximate) answer from the graph with a fair degree of confidence.On the other hand, a quadratic graphs as a wiggly parabola. If you plot a few non-x-intercept points and then draw a curvy line through them, how do you know if you got the x-intercepts even close to being correct? You don't. The only way you can be sure of your x-intercepts is to set the quadratic equal to zero and solve. But the whole point of this topic is that they don't want you to do the (exact) algebraic solving; they want you to guess from the pretty pictures. So "solving by graphing" tends to be neither "solving" nor "graphing". That is, you don't actually graph anything, and you don't actually do any of the "solving". Instead, you are told to punch some buttons on your graphing calculator and look at the pretty picture, and then you're told which other buttons to hit so the software can compute the contents back next
  • 61. intercepts (or you're told to guess from the pretty picture in the book, hoping that the printer lined up the different print runs for the different ink colors exactly right). I think the educators are trying to "help" you "discover" the connection between x-intercepts and solutions, but the concept tends to get lost in all the button-pushing. Okay, enough of my ranting... To "solve" by graphing, the book may give you a very neat graph, probably with at least a few points labeled; the book will ask you to state the points on the graph that represent solutions. Otherwise, it will give you a quadratic, and you will be using your graphing calculator to find the answer. Since different calculator models have different key-sequences, I cannot give instruction on how to "use technology" to find the answers, so I will only give a couple examples of how to solve from a picture that is given to you. • Solve x2 – 8x + 15 = 0 by using the following graph. The graph is of the related quadratic, y = x2 – 8x + 15, with the x-intercepts being where y = 0. The point here is to look at the picture (hoping that the points really do cross at whole numbers, as it appears), and read the x- intercepts (and hence the solutions) from the picture. The solution is x = 3, Since x2 – 8x + 15 factors as (x – 3)(x – 5), we know that our answer is correct. contents back next
  • 62. • Solve 0.3x2 – 0.5x – 5/3 = 0 by using the following graph. For this picture, they labeled a bunch of points. Partly, this was to be helpful, because the x-intercepts are messy (so I could not have guessed their values without the labels), but mostly this was in hopes of confusing me, in case I had forgotten that only the x-intercepts, not the vertices or y-intercepts, correspond to "solutions". The x-values of the two points where the graph crosses the x-axis are the solutions to the equation. The solution is x = –5/3, 10/ 3 • Find the solutions to the following quadratic: contents back next
  • 63. They haven't given me the quadratic equation, so I can't check my work algebraically. (And, technically, they haven't even given me a quadratic to solve; they have only given me the picture of a parabola from which I am supposed to approximate the x-intercepts, which really is a different question....) I ignore the vertex and the y-intercept, and pay attention only to the x-intercepts. The "solutions" are the x- values of the points where the pictured line crosses the x-axis: The solution is x = –5.39, 2.76 "Solving" quadratics by graphing is silly in "real life", and requires that the solutions be the simple factoring- type solutions such as "x = 3", rather than something like "x = –4 + sqrt(7)". In other words, they either have to "give" you the answers (by labeling the graph), or they have to ask you for solutions that you could have found easily by factoring. About the only thing you can gain from this topic is reinforcing your understanding of the connection between solutions and x-intercepts: the solutions to "(some polynomial) equals (zero)" correspond to the x-intercepts of "y equals (that same polynomial)". If you come away with an understanding of that concept, then you will know when best to use your graphing calculator or other graphing software to help you solve general polynomials; namely, when they aren't factorable. contents back next
  • 64. Name: ___________________ Section: _______ Instructor: ________________ Date: _______ Rating: ______ Instruction: Solve each equation by graphing. 1. x2 – 6x + 9 = 0 _____________________________________________________ 2. x2 – 5x + 10 = 0 _____________________________________________________ 3. 2x2 – 6x + 8 = 0 _____________________________________________________ 4. x2 – 7x + 12 = 0 _____________________________________________________ 5. 2x2 – 8x + 10 = 0 _____________________________________________________ 6. 3x2 + 6x – 9 = 0 _____________________________________________________ 7. x2+ 8x – 12 = 0 _____________________________________________________ contents back next
  • 65. 8. 2 + 4x – 3 = 0 _____________________________________________________ 9. x2 – 2x – 2 = 0 _____________________________________________________ 10. 2x2 – 4x – 2 = 0 _____________________________________________________ 11. 4x2 – 8x – 16 = 0 _____________________________________________________ 12. x2 – 9x + 21 = 0 _____________________________________________________ 13. x2 + 10x + 18 = 0 _____________________________________________________ 14. 2x2 – 16x + 8 = 0 _____________________________________________________ 15. 3x2 – 12x – 9 = 0 _____________________________________________________ contents back next
  • 66. A. Solve by factoring. 1. x2 – 3x – 10 = 0 2. x2 + 2x = 8 3. x2 – x – 4 = 2 4. 2x2 – 6x – 36 = x2 – 15 5. 4x2 + 4x = 15 6. 6x2 + 11x – 2 = 8 7. 49x2 + 28x – 10 = 0 8. 6x4 – 4x3 – 10x2 = 0 9. 18 + 15x – 18x2 = 0 10. x4 – 4x2 + 3 = 0 B. Solve by completing the square. 1. x2 - 4x – 3 = 0 2. x2 + 3x – 6 = 0 3. x2 – 7x + 5 = 0 4. 2x2 + 5x + 1 = 0 5. 2x2 + 8x – 5 = 0 C. Solve for x by the quadratic formula. 1. x2.- 4x – 7 = 0 2. x2 – 3x + 4 = 0 3. 2x2 + 4x + 5 = 0 4. x2 + 7x – 3 = 0 5. x2 – 7x + 2 = 0 6. x2 + 5x – 7 = 0 7. x2 + 9x – 3 = 0 8. 4x2 – 6x + 2 = 0 9. 9x2 – 9x – 10 = 0 10. x2 + 5x + 8 = 0 contents back next
  • 67. Chapter IV Much of the study in quadratic equation consist of different solving equation, we have equation in quadratic form, equation containing radicals and equation reducible to quadratic equation. They have their own steps and procedures to be followed in order to solve the given equation. TARGET SKILLS: At the end of this chapter, students are expected to: • discuss solving equation on quadratic; • determine the index and its radicals; • interpret the solution of the original equation; and • select appropriate method in solving quadratic equation. contents back next
  • 68. Lesson 10 Equation in Quadratic Form OBJECTIVES: At the end of this lesson, students are expected to: •identify equation in quadratic form; •select appropriate method in solving quadratic equation; and •change the equation in standard form. Quadratic in Form An equation is quadratic in form when it can be written in this standard form where the same expression is inside both ( )'s. In other words, if you have a times the square of the expression following b plus b times that same expression not squared plus c equal to 0, you have an equation that is quadratic in form. If we substitute what is in the ( ) with a variable like t, then the original equation will become a quadratic equation. contents back next
  • 69. Solving Equations that are Quadratic in Form Step 1: Write in Standard Form, , if needed. If it is not in standard form, move any term(s) to the appropriate side by using the addition/subtraction property of equality. Also, make sure that the squared term is written first left to right, the expression not squared is second and the constant is third and it is set equal to 0. Step 2: Substitute a variable in for the expression that follows b in the second term. In other words, substitute your variable for what is in the ( ) when it is in standard form, . I’m going to use t for my substitution, but really you can use any variable as long as it is not the variable that is used in the original equation. Step 3: Solve the quadratic equation created in step 2. You can use any method you want to solve the quadratic equation: factoring, completing the square or quadratic formula. Step 4: Find the value of the variable from the original equation. Keep in mind that you are finding a solution to the original equation and that the variable you substituted in for in step 2 is not your original variable. Use the substitution that was used to set up step 2 and then solve for the original variable. contents back next
  • 70. . Step 5: Check your solutions. In some cases, you will be working with rational exponents and square roots in your problems. Those types of equations can cause extraneous solutions. Recall that an extraneous solution is one that is a solution to an equation after doing something like raising both sides of an equation by an even power, but is not a solution to the original problem. Even though not all of the quadratic in form equations can cause extraneous solutions, it is better to be safe than sorry and just check them all. Example 1: Solve the equation that is quadratic in form: Standard Form, *Rewriting original equation to show it is quadratic in form *Note that (y squared) squared = y to the fourth *When in stand. form, let t = the expression following b. Next, we need to substitute t in for y squared in the original equation. *Original equation *Substitute t in for y squared contents back next
  • 71. Note how we ended up with a quadratic equation when we did our substitution. From here, we need to solve the quadratic equation that we have created. Solve the quadratic equation: factoring, completing the square or quadratic formula. *Factor the trinomial *Use Zero-Product Principle *Set 1st factor = 0 and solve *Set 2nd factor = 0 and solve contents back next
  • 72. Let's find the value(s) of y when t = -4: *Plug in - 4 for t *Use square root method to solve for y *First solution *Second solution Let's find the value(s) of y when t = 1: *Plug in 1 for t *Use square root method to solve for y *First solution *Second solution contents back next
  • 73. . Example 2: Solve the equation that is quadratic in form: Standard Form, *Inverse of add. 3 is sub. 3 *Equation in standard form Note how when you square x to the 1/3 power you get x to the 2/3 power, which is what you have in the first term. * Rewriting original equation to show it is quadratic in form *Note that (x to the 1/3 power) squared = x to the 2/3 power *When in stand. form, let t = the expression following b. Next, we need to substitute t in for x to the 1/3 power in the original equation. contents back next
  • 74. *Original equation *Substitute t in for x to the 1/3 power You can use any method you want to solve the quadratic equation: factoring, completing the square or quadratic formula. *Factor the trinomial *Use Zero-Product Principle *Set 1st factor = 0 and solve *Set 2nd factor = 0 and solve contents back next
  • 75. Let's find the value(s) of x when t = 3: *Plug in 3 for t *Solve the rational exponent equation *Inverse of taking it to the 1/3 power is raising it to the 3rd power Let's find the value(s) of x when t = -1: *Plug in -1 for t *Solve the rational exponent equation *Inverse of taking it to the 1/3 power is raising it to the 3rd power Let's double check to see if x = 27 is a solution to the original equation. *Plugging in 27 for x *True statement contents back next
  • 76. Since we got a true statement, x = 27 is a solution. Let's double check to see if x = -1 is a solution to the original equation. *Plugging in -1 for x *True statement Since we got a true statement, x = -1 is a solution. There are two solutions to this equation: x = 27 and x = -1. Exercises: 1. a4 + 2a2 – 5 = 0 2. x2 – 3x + 2 = 0 3. s6 + 8s3 – 6 = 0 4. n2 – 6n + 10 = 0 5. g8 + 2g4 – g = 0 contents back next
  • 77. Name: ___________________ Section: _______ Instructor: ________________ Date: _______ Rating: ______ Instruction: Solve the equation that is in quadratic form. 1. a8 + 2a4 – 8 = 0 _____________________________________________________ 2. l2 + 4l2 – 6 = 0 _____________________________________________________ 3. e4 – 8e2 – 3 = 0 _____________________________________________________ 4. l6 – 10l – 5 = 0 _____________________________________________________ 5. i10 – 8i5 – 4 = 0 _____________________________________________________ 6. s6 – 5s3 – 25 = 0 _____________________________________________________ contents back next
  • 78. 7. h2/4 + 8h1/4 – 12 = 0 _____________________________________________________ 8. a6- 5a4 – 15 = 0 _____________________________________________________ 9. n8 + 12n2 – 8 = 0 _____________________________________________________ 10. e9 – 3n3 – 10 = 0 _____________________________________________________ 11. x2/3 – 2x 1/3 = 8 _____________________________________________________ 12. x3/6 – 3x1/2 = 9 _____________________________________________________ 13. y2- 8y = 5 _____________________________________________________ 14. y4 + 2y2 = 6 _____________________________________________________ 15. x6 – 9x2 + 8 = 0 _____________________________________________________ contents back next
  • 79. Lesson 11 Equation Containing Radicals OBJECTIVES: At the end of this lesson, students are expected to: •determine the index and its radicals; •positively respond to the note to be remembered; and •perform isolation of one radical if there are two radicals in the equation. In the radicals which is read the “nth root of b,” the positive integer n is called the index or order of the radical, and b is called its radicand. When n is 2, 2 is no longer written, just simply write instead of to indicate the square root of b, thus is read as “cube root of b”; as “4 th of b”. Note: • In order to solve for x, you must isolate x. • In order to isolate x, you must remove it from under the radial. • If there are two radicals in the equation, isolate one of the radicals. • Then raise both sides of the equation to a power equal to the index of the isolated radical. • Isolate the the remaining radical. • Raise both sides of the equation to a power equal to the index of the isolated radical. • You should now have a polynomial equation. Solve it. • Remember that you did not start out with a polynomial; therefore, there may be extraneous solutions. Therefore, you must check your answers. contents back next
  • 80. Example 1: First make a note of the fact that you cannot take the square root of a negative number. Therefore, the term is valid only if and the second term is valid if Isolate the term Square both sides of the equation. Isolate the term Square both sides of the equation. Check the solution by substituting 9 in the original equation for x. If the left side of the equation equals the right side of the equation after the substitution, you have found the correct answer. contents back next
  • 81. • Left side: •Right Side:1 • Since the left side of the original equation does not equal the right side of the original equation after we substituted our solution for x, then there is no solution. You can also check the answer by graphing the equation: .The graph represents the right side of the original equation minus the left side of the original equation.. The x- =5 intercept(s) of this graph is (are) the solution(s). Since there are no x-intercepts, there are no solutions. Exercises: Solve each of the following equation. 1. =3 2. =x+1 3. = 4. =5 5. = 10 contents back next
  • 82. Name: ___________________ Section: _______ Instructor: ________________ Date: _______ Rating: ______ Instruction: Solve each of the following equation. 1. _____________________________________________________ 2. _____________________________________________________ 3. _____________________________________________________ 4. _____________________________________________________ 5. _____________________________________________________ 6. _____________________________________________________ contents back next
  • 83. 7. ____________________________________________________ 8. ____________________________________________________ 9. ____________________________________________________ 10. ____________________________________________________ 11. ____________________________________________________ 12. ____________________________________________________ 13. ____________________________________________________ 14. ____________________________________________________ 15. ____________________________________________________ contents back next
  • 84. Lesson 12 Equations Reducible to Quadratic Equations OBJECTIVES: At the end of this lesson, students are expected to: •interpret the solution of the original equation; •organize the equation if it is quadratic equation; and •solve the equation by factoring or quadratic formula. A variety of equations can be transformed into quadratic equations and solved by methods that we have discussed in the previous section. We will consider fractional equations, equations involving radicals and equation that can be transformed into quadratic equations by appropriate substitutions. Since the transformation process may introduce extraneous roots which are not solutions of the original equation, we must always check the solution in the original equation. Example: Solve 1 1 7 x+2 + x+3 = 12 Solution: First note that neither -2 nor -3 can be a solution since at either of these points the equation is meaningless. Multiplying by the LCD, 12(x+2) (x+3), we get 12(x+3) + 12(x+2) = 7(x+2) (x+3) 24x + 60 = 7(x2 + 5x + 6) or 7x2 + 11x – 18 = 0 Factoring, we get, (7x + 18)(x – 1) = 0 contents back next
  • 85. x = 1 or -18 7 If x = 1, _1_ _1_ _1_ _1_ _7_ 1+2 1+3 = 3 + 4 = 12 Therefore x = 1 is a solution. If x = -18, __1__ + __1__ 7 -18/7 + 2 -18/7 + 3 = __7__ + __7__ -18 + 14 -18 + 21 = _-7_ + _7_ = _7_ 4 3 12 Therefore, x = _-18_ is a solution. 7 Example 2. = -2 Solution: squaring both sides of the equation, we obtain, = +4 2x – 16 = 4 contents back next
  • 86. Dividing both sides by 2 gives, x – 8 = -2 Squaring both sides of the equation we get 2 (x - 8) = x2 - 16x + 64 = 4 (x +16) x2 – 20x = 0 x(x – 20) =0 x = 0 or x = 20 Check: if x = 0, = = 8=6–2 8≠4 Therefore x = 20 is not a solution of the original equation. Thus the only root of Many equations are not quadratics equations. However, we can transform them by means of appropriate substitutions into quadratics equations and then solve these by techniques that we know. contents back next
  • 87. Example: Solve: 1. 2x-2 – 7x-1 + 3 = 0 2. x4 – 2x2 – 2 = 0 3. 2 – Solutions a. Let u = x-1. Then u2 = (x-1)2 = x-2 and our equation becomes 2u2 – 7u + 3 = 0, a quadratic equation in u. To solve the equation, we factor the left-hand side. (2u – 1)( u – 3) = 0 U = ½ or u = 3 Since u = x-1, x-1= or x-1 = 3, from which x = 2 or x = Check: if x = 2, 2(2-2) – 7(2-1) + 3 = Thus x = 2 is solution If x = 1/3, 2(1/3)-2 – 7(1/3)-1 + 3 = 2(3)2 – 7(3) + 3 So, x = 1/3 is a solution. b. Let u = x2. Then u2 = x4 and the given equation becomes a quadratic equation in u. u2 – 2u – 2 = 0 contents back next
  • 88. u= = . u=1+ or u = Since u = x2 and u = < 0, we have to discard this solution. . u = x2 = implies x= It is simple to verify that both values of x satisfy the original equation. The roots of x4 – x2 – 2 = 0 are and - c. Let u = .This substitution yields a quadratic equation in u. u2 – u – 2 = 0 (u – 2)(u + 1) = 0 u = 2 0r u = ˉ1 u= =2 implies x = 2(4x + 1) or x= u= =1 implies x = ˉ4x – 1 or x= Again, it can easily be verified that both solutions check in the original equation. The roots are and contents back next
  • 89. Name: ___________________ Section: _______ Instructor: ________________ Date: _______ Rating: ______ Instruction: Solve the following equation. 1. _____________________________________________________ 2. _____________________________________________________ 3. _____________________________________________________ 4. _____________________________________________________ 5. _____________________________________________________ 6. _____________________________________________________ contents back next
  • 90. 7. _____________________________________________________ 8. _____________________________________________________ 9. _____________________________________________________ 10. _____________________________________________________ 11. _____________________________________________________ 12. _____________________________________________________ 13. _____________________________________________________ 14. _____________________________________________________ 15. _____________________________________________________ contents back next
  • 91. A. Solve for x. C. Solve for x. 1. + = 1. =4 2. -5=0 2. + + =0 3. = 3. + = 4. = 5. =x+2 4. + =2 B. Reduce to quadratic equation. 5. + = 1. x4 – 5x + 4 = 0 2. 4(x + 3) + 5 = 21 3. x2/3 – 5x1/3 – 6 = 0 4. (x2 + 4x)2 – (x2 + 4x) = 20 5. 2x4 – 9x2 + 7 = 0 contents back next
  • 92. Chapter V The discriminant gives additional information on the nature of the roots beyond simply whether there are any repeated roots: it also gives information on whether the roots are real or complex, and rational or irrational. More formally, it gives information on whether the roots are in the field over which the polynomial is defined, or are in an extension field, and hence whether the polynomial factors over the field of coefficients. This is most transparent and easily stated for quadratic and cubic polynomials; for polynomials of degree 4 or higher this is more difficult to state. TARGET SKILLS: At the end of this chapter, students are expected to: •determine discriminant, roots and coefficient; • discuss the relation the roots and coefficient; • find the sum and product of the roots; and • change quadratic equation to discriminant formula. contents back next
  • 93. Lesson 13 The Discriminant and the roots of a Quadratic Equation OBJECTIVES: At the end of this lesson, students are expected to: •determine discriminant and the roots; •compare discriminant and the nature of the roots; and •change quadratic equation to discriminant using the nature of the roots. Example 1. Find the x-intercept of y = 3x² - 6x + 4. Solution: As already mentioned, the values of x for which 3x² - 6x + 4 = 0 give the x-intercepts of the function. We apply the quadratic formula in solving the equation. 3x² - 6x + 4 = 0 x= = Since is not a real number, the equation 3x² - 6x + 4 = 0 has no real root. This means that the parabola y = 3x² - 6x + 4 does not intersect the x-axis. Let us write the equation in the form y = a(x – h)²+ k. y = 3(x² – 2x)² + 4 = 3(x – 1)² + 1 contents back next
  • 94. ∆ = b² - 4ac Roots of ax² + bx + c = 0 Positive Real and distinct r= s= Zero Real and equal r=s= Negative No real roots Example 2. Use the disciminant to determine the nature of the roots of the following quadratic equation. a. x² - x + ¼ = 0 a = 1, b = ˉ1, c = ¼ b² - 4ac = (ˉ1)² - 4 (1)(¼) =1–1 =0 There is only one solution, that is, a double root. Note that x² - x = ½ = (x - ½), so that double root is ˉb/2a = ½. b. 5x² - 4x + 1 = 0 a = 5, b = ˉ4, c = 1 b² - 4ac = (ˉ4)² - 4 (5)(1) = 16 – 20 = ˉ4 < 0 There are no real roots since a negative number has no real square root. contents back next
  • 95. Name: ___________________ Section: _______ Instructor: ________________ Date: _______ Rating: ______ Instruction: Use the Discriminant to determine the nature of the root of the following Quadratic Equations. 1.x2- 2x – 3=0 _____________________________________________________ 2. 6x2 – x – 1 = 0 _____________________________________________________ 3. 2x2 – 50 = 0 _____________________________________________________ 4. x2 – 8x + 12 = 0 _____________________________________________________ 5. x2 + 5x – 14 = 0 _____________________________________________________ 6. -4x2 – 4x + 1 = 0 _____________________________________________________ 7. 7x2 + 2x – 1 = 0 _____________________________________________________ contents back next
  • 96. 8.x2 + 3x = 40 _____________________________________________________ 9. 3x2= 5x – 1 _____________________________________________________ 10. 3x2+ 12 – 1=0 _____________________________________________________ 11. (x-2)(x-3) = 4 _____________________________________________________ 12.2x2 + 2x + 1 = 0 _____________________________________________________ 13. 7x2 + 3 – 6x = 0 _____________________________________________________ 14. 5x2 – 6x + 4 = 0 _____________________________________________________ 15. 3x2 + 2x + 2 = 0 _____________________________________________________ contents back next
  • 97. Lesson 14 Relation between roots and coefficient OBJECTIVES: At the end of this lesson, students are expected to: •classify roots and coefficient; •discuss relations between the roots and coefficient of the quadratic equation; and •find the sum and product of the roots of a given quadratic equation There are some interesting relations between the sum and the product of the roots of a quadratic equation. To discover these, consider the quadratic equation ax2 + bx + c = 0, where a ≠ 0. Multiply both sides of this equation by 1/a so that the coefficient of x2 is 1. (ax2 + bx + c) = We obtain an equivalent quadratic equation in the form x2 + + =0 If r and s are the roots of the quadratic equation ax2 + bx + c = 0, then from the quadratic formula r= and s = contents back next
  • 98. Adding the roots, we obtain r+s= + = = Multiplying the roots, we obtain rs = = c/a Observe the coefficient in the quadratic equation x2 + bx/a + c/a = 0. How do they compare with the sum and the product of the roots? Did you observe the following? 1. The sum of the roots is equal to the negative of the coefficient of x. r + s = -b / a 2. The product of the roots is equal to the constant term rs = c / a An alternate way of arriving at these relations is as follows Let r and s be the roots of x2 + bx/a + c/a = 0. Then x - r)(x – s) = 0 Expanding gives, x2 – rx – sx + rs = 0 or x2 – (r + s)x + rs = 0 contents back next
  • 99. Comparing the coefficients of the corresponding terms, we obtain r + s = -b / a and rs c / a The above relations between the roots and the coefficients provide a fast and convenient means of checking the solutions of a quadratic equation. Example: Solve and check. 2x2 + x – 6 = 0 Solutions: 2x2 + x – 6 = (2x – 3)(x + 2) = 0 x = 3/2 or x = 2 The roots are 3/2 and 2. To check, we add the roots, 3/2 = (-2) = -1/2 = -b/a. and multiply them 3/2 = (-2) = -3 = c/a Example: Find the sum and the product of the roots of 3x2 – 6x + 8 = 0 without having to first determine the roots. Solution: The sum of the roots is r + s = -c/a = -(-6)/3 = 2 and their product is rs = c/a = 8/3 contents back next
  • 100. Name: ___________________ Section: _______ Instructor: ________________ Date: _______ Rating: ______ Instruction: Without solving the roots, find the sum and product of the roots of the following. 1. 6x2 – 5x + 2 = 0 _____________________________________________________ 2. x2 + x – 182 = 0 _____________________________________________________ 3. x2 – 5x – 14 = 0 _____________________________________________________ 4. 2x2 – 9x + 8 = 0 _____________________________________________________ 5. 3x2 - 5x – 2 = 0 _____________________________________________________ 6. x2 – 8x – 9 = 0 _____________________________________________________ contents back next
  • 101. 7. 2x2 – 3x – 9 = 0 _____________________________________________________ 8. x2 + x – 2 - _____________________________________________________ 9. 3x2 + 2x – 8 = 0 _____________________________________________________ 10. 16x2 – 24x + ½ = 0 _____________________________________________________ 11. x2 – 6x + 25 = 0 _____________________________________________________ 12. 3x2 + x – 2 = 0 _____________________________________________________ 13. 5x2 + 11x – 8 = 0 _____________________________________________________ 14. x2 – 8x + 16 = 0 _____________________________________________________ 15. 4x2 – 16x + 10 = 0 _____________________________________________________ contents back next
  • 102. A. Use the discriminant to determine which of the following quadratic equations have two, one or no real roots. Give reasons. 1. x2 – 5x – 5 = 0 2. x2 – 3x – 2 = 3x – 11 3. 5x2 – 9x = 2x – 7 4. 2x2 – 7x + 8 = 0 5. 3 – 4x – 2x2 = 0 6. 4x2 – 9x + 5 = 3x – 7 B. By using the relations between roots and coefficients, determine if the given #s are roots of the corresponding given equation. 1. 6x2 – 5x + 3 = 0 (1/6, -1) 2. x2 + x _ 182 = 0 (13, -4) 3. x2 – 5x – 14 =0 (2, -7) 4. 2x – 9x + 8 = 0 (3, 4/3) 5. 3x2 – 5x – 2 = 0 (2, -1/3) C. Given one roots of the equation, find the other. 1. x2 – 8x – 9 = 0; r= 1 2. 2x2 – 3x – 9 = 0; r= 3 3. x2 + x – 2 - =0 r= contents back next
  • 103. Chapter VI In this chapter, students will be taught how to find solutions to quadratic equations. This lesson assumes students are already familiar with solving simple quadratic equations by hand, and that they have become relatively comfortable using their graphing calculator for solving arithmetic problems and simple algebra problems. Students will also be shown strategies on how to use the keys on the graphing calculator to show a complete graph. TARGET SKILLS: At the end of this chapter, students are expected to: • use calculator in solving quadratic equation; • solve equation on a calculator; and. • improve skills using calculator by solving quadratic equation. contents back next